+ All Categories
Home > Documents > Peripheral Vision - Amazon Simple Storage Serviceweb)FINAL.pdf · THE OFFICIAL CME PUBLICATION OF...

Peripheral Vision - Amazon Simple Storage Serviceweb)FINAL.pdf · THE OFFICIAL CME PUBLICATION OF...

Date post: 30-Mar-2019
Category:
Upload: nguyenxuyen
View: 215 times
Download: 0 times
Share this document with a friend
32
THE OFFICIAL CME PUBLICATION OF THE AMERICAN COLLEGE OF EMERGENCY PHYSICIANS Volume 30 Number 4 April 2016 Mindful Medicine The scope of psychiatric disorders and the use of medications to treat them has surged in recent years, breeding a new generation of atypical antipsychotics. Emergency physicians must be familiar with the efficacy, optimal use, and untoward effects of these powerful agents when used for chemical restraint and the management of behavioral problems. Peripheral Vision The use of ultrasound-guided regional anesthesia is rapidly emerging as an alternative to anatomic and nerve stimulation block techniques. When administered in appropriate clinical scenarios, they can provide excellent — even superior — analgesia with a lower risk of adverse side effects than other types of pain relief, including opiate medication and procedural sedation.
Transcript

THE OFFICIAL CME PUBLICATION OF THE AMERICAN COLLEGE OF EMERGENCY PHYSICIANS

Volume 30 Number 4 April 2016

Lumbar puncture (LP) is used in the diagnostic evaluation of central nervous system (CNS) processes, most commonly in cases of suspected infection and subarachnoid hemorrhage. Less commonly, the procedure is used for therapeutic purposes (eg, in cases of idiopathic intracranial hypertension).

Mindful MedicineThe scope of psychiatric disorders and the use of medications to treat them has surged in recent years, breeding a new generation of atypical antipsychotics. Emergency physicians must be familiar with the efficacy, optimal use, and untoward effects of these powerful agents when used for chemical restraint and the management of behavioral problems.

Peripheral VisionThe use of ultrasound-guided regional anesthesia is rapidly emerging as an alternative to anatomic and nerve stimulation block techniques. When administered in appropriate clinical scenarios, they can provide excellent — even superior — analgesia with a lower risk of adverse side effects than other types of pain relief, including opiate medication and procedural sedation.

Contributor Disclosures. In accordance with the ACCME Standards for Commercial Support and policy of the American College of Emergency Physicians, all individuals with control over CME content (including but not limited to staff, planners, reviewers, and authors) must disclose whether or not they have any relevant financial relationship(s) to learners prior to the start of the activity. These individuals have indicated that they have a relationship which, in the context of their involvement in the CME activity, could be perceived by some as a real or apparent conflict of interest (eg, ownership of stock, grants, honoraria, or consulting fees), but these individuals do not consider that it will influence the CME activity. Sharon E. Mace, MD, FACEP; Baxter Healthcare, consulting fees, fees for non-CME services, and contracted research; Gebauer Company, contracted research; Halozyme, consulting fees. Joshua S. Broder, MD, FACEP; GlaxoSmithKline; his wife is employed by GlaxoSmithKline as a research organic chemist. All remaining individuals with control over CME content have no significant financial interests or relationships to disclose.

Method of Participation. This educational activity consists of two lessons, a post-test, and evaluation questions; as designed, the activity it should take approximately 5 hours to complete. The participant should, in order, review the learning objectives, read the lessons as published in the print or online version, and complete the online post-test (a minimum score of 75% is required) and evaluation questions. Release date March 1, 2016. Expiration date February 29, 2019.

Accreditation Statement. The American College of Emergency Physicians is accredited by the Accreditation Council for Continuing Medical Education to provide continuing medical education for physicians.

The American College of Emergency Physicians designates this enduring material for a maximum of 5 AMA PRA Category 1 Credits™. Physicians should claim only the credit commensurate with the extent of their participation in the activity.

Each issue of Critical Decisions in Emergency Medicine is approved by ACEP for 5 ACEP Category I credits. Approved by the AOA for 5 Category 2-B credits.

Commercial Support. There was no commercial support for this CME activity.Target Audience. This educational activity has been developed for emergency physicians.

Critical Decisions in Emergency Medicine is a trademark owned and published monthly by the American College of Emergency Physicians, PO Box 619911, Dallas, TX 75261-9911. Send address changes and comments to Critical Decisions in Emergency Medicine, PO Box 619911, Dallas, TX 75261-9911, or to [email protected]; call toll-free 800-798-1822, or 972-550-0911.

Copyright 2016 © by the American College of Emergency Physicians. All rights reserved. No part of this publication may be reproduced, stored, or transmitted in any form or by any means, electronic or mechanical, including storage and retrieval systems, without permission in writing from the Publisher. Printed in the USA.

The American College of Emergency Physicians (ACEP) makes every effort to ensure that contributors to its publications are knowledgeable subject matter experts. Readers are nevertheless advised that the statements and opinions expressed in this publication are provided as the contributors’ recommendations at the time of publication and should not be construed as official College policy. ACEP recognizes the complexity of emergency medicine and makes no representation that this publication serves as an authoritative resource for the prevention, diagnosis, treatment, or intervention for any medical condition, nor should it be the basis for the definition of, or standard of care that should be practiced by all health care providers at any particular time or place. Drugs are generally referred to by generic names. In some instances, brand names are added for easier recognition. Device manufacturer information is provided according to style conventions of the American Medical Association. ACEP received no commercial support for this publication.

To the fullest extent permitted by law, and without limitation, ACEP expressly disclaims all liability for errors or omissions contained within this publication, and for damages of any kind or nature, arising out of use, reference to, reliance on, or performance of such information.

Lesson 7 n Ultrasound-Guided Regional Anesthesia . . . . . . . . 3

LLSA Literature Review . . . . . . . . . . . . . . . . . . . . . . . . . . . . . . . . . . . 13

Critical Procedure . . . . . . . . . . . . . . . . . . . . . . . . . . . . . . . . . . . . . . . . 14

Critical ECG . . . . . . . . . . . . . . . . . . . . . . . . . . . . . . . . . . . . . . . . . . . . . 16

Lesson 8 n Second-Generation Antipsychotics . . . . . . . . . . . . . . 17

Critical Image . . . . . . . . . . . . . . . . . . . . . . . . . . . . . . . . . . . . . . . . . . . 28

CME Questions . . . . . . . . . . . . . . . . . . . . . . . . . . . . . . . . . . . . . . . . . . 30

Drug Box/Tox Box . . . . . . . . . . . . . . . . . . . . . . . . . . . . . . . . . . . . . . . 32

Critical Decisions in Emergency Medicine is the official CME publication of the American College of Emergency Physicians. Additional volumes are available to keep emergency medicine professionals up to date on relevant clinical issues.

EDITOR-IN-CHIEFLouis G. Graff, IV, MD, FACEP

University of Connecticut, Farmington, CT [email protected]

EDITOR-ELECTMichael S. Beeson, MD, MBA, FACEP

Northeastern Ohio Universities, Rootstown, OH

SECTION EDITORSJ. Stephen Bohan, MD, MS, FACEP

Brigham & Women’s Hospital/Harvard Medical School, Boston, MA

Joshua S. Broder, MD, FACEP Duke University, Durham, NC

Amal Mattu, MD, FACEP University of Maryland, Baltimore, MD

Lynn P. Roppolo, MD, FACEP University of Texas Southwestern Medical Center,

Dallas, TX

Christian A. Tomaszewski, MD, MS, MBA, FACEP University of California Health Sciences, San Diego, CA

Steven J. Warrington, MD Kaweah Delta Medical Center, Visalia, CA

ASSOCIATE EDITORSWalter L. Green, MD, FACEP

University of Texas Southwestern Medical Center, Dallas, TX

John C. Greenwood, MD University of Pennsylvania, Philadelphia, PA

Daniel A. Handel, MD, MPH, FACEP Medical University of South Carolina, Charleston, SC

Frank LoVecchio, DO, MPH, FACEP Maricopa Medical Center/Banner Phoenix Poison

and Drug Information Center, Phoenix, AZ

Sharon E. Mace, MD, FACEP Cleveland Clinic Lerner College of Medicine/Case

Western Reserve University, Cleveland, OH

Jennifer L. Martindale, MD SUNY Downstate Medical Center/

Kings County Hospital Center, Brooklyn, NY

Robert A. Rosen, MD, FACEP University of Virginia, Charlottesville, VA

George Sternbach, MD, FACEP Stanford University Medical Center, Stanford, CA

Kathleen Wittels, MD Brigham & Women’s Hospital/

Harvard Medical School, Boston, MA

RESIDENT EDITORNathaniel Mann, MD

University of Cincinnati Medical Center, Cincinnati, OH

EDITORIAL STAFFRachel Donihoo, Managing Editor

[email protected] Hamilton, Educational Products Assistant

Lexi Schwartz, Subscriptions CoordinatorMarta Foster, Director, Educational Products

ISSN2325-0186(Print) ISSN2325-8365(Online)

n Pediatric Pain Management

n Concussion

I N T H I S I S S U E

N E X T M O N T H

April 2016 n Volume 30 Number 4 3

FROM THE EM MODEL19.0 Procedures and Skills Integral to the Practice of Emergency Medicine

19.3 Anesthesia and Acute Pain Management

LESSON 7

By John Eicken, MD, and Joshua Rempell, MD, MPHDr. Eicken and Dr. Rempell are faculty physicians in the Department of Emergency Medicine, Division of Emergency Ultrasound, at Brigham and Women’s Hospital in Boston, Massachusetts.

Reviewed by Kathleen A. Wittels, MD

n What are the indications and contraindications for placement of an ultrasound-guided peripheral nerve block?

n What are the procedure’s most important risks and benefits?

n What medications and needles are required for the performance of ultrasound-guided regional anesthesia?

n What general principles and techniques should be employed during the procedure?

n Which specific peripheral nerve blocks are of greatest utility in the emergency department, and how should the placement of each type be approached?

CRITICAL DECISIONSOBJECTIVESOn completion of this lesson, you should be able to:

1. Identify patients and clinical scenarios that are appropriate for placement of an ultrasound-guided peripheral nerve block in the emergency department.

2. Describe point-of-care principles that promote safe and accurate performance of an ultrasound-guided peripheral nerve block.

3. List the most common local anesthetic agents used to perform ultrasound-guided peripheral nerve blocks and understand their unique characteristics.

4. Understand the advantages and benefits of ultrasound-guided peripheral nerve blocks compared to standard care.

Ultrasound-Guided Regional Anesthesia

Peripheral Vision

Critical Decisions in Emergency Medicine4

CASE PRESENTATIONS■ CASE ONE

An 85-year-old woman with a history of chronic obstructive pulmonary disease and coronary artery disease presents with left hip and groin pain after falling at home. She lost her balance while gardening and fell on the grass, landing on the left side of her body. When EMS arrived, they found the patient awake, alert, oriented, and lying on the ground.

In the emergency department, she has normal vital signs and reports pain at a level of 9 out of 10 in her left hip and groin, which she describes as sharp, non-radiating, and worse with movement. Her physical examination reveals a shortened and externally rotated left lower extremity. The skin overlying her hip and groin is intact and without ecchymosis, erythema, or edema. There are normal and equal femoral, posterior tibial, and dorsalis pedis pulses bilaterally. The patient exhibits 5 out of 5 strength with plantar and dorsiflexion of the left foot; her

sensation to touch is intact. Any movement, passive or active, results in severe pain in the left hip.

■ CASE TWOA 28-year-old man who is

otherwise healthy presents with a laceration over the plantar aspect of his right foot. He inadvertently stepped on the sharp edge of a rock while running in bare feet. In the emergency department, the patient’s vital signs are within normal limits and hemostasis of the laceration has been achieved following direct pressure from a towel. He reports an 8 out of 10 pain level overlying the wound that is worse with ambulation; he denies any other injuries or areas of discomfort. His tetanus vaccination is up to date.

A physical examination reveals a 4-cm linear laceration that extends from the base of his third right toe toward the middle of the sole. There is no active bleeding. The patient

Many of the conditions and ailments that prompt individuals to seek care in emergency departments are associated with pain. Emergency medicine providers, who are called upon to treat patient discomfort in a timely, safe, and effective manner, commonly rely on oral, intravenous, and subcutaneous medications. The use of ultrasound-guided regional anesthesia is rapidly emerging as an alternative to anatomic and nerve stimulation block techniques.

Point-of-care ultrasonography (POCUS) has become increasingly utilized in the emergent setting for both diagnostic and therapeutic purposes; its ability to supply real-time feedback and guidance can enhance accuracy and precision during invasive, lifesaving procedures. When administered in appropriate clinical scenarios, peripheral nerve blocks can provide excellent — even superior — analgesia with a lower risk of adverse side effects than other types of pain relief, including opiate medication and procedural sedation.

Background POCUS is increasingly being used in

emergency departments during the per-formance of invasive procedures, includ-ing central venous catheter placement, thoracentesis, paracentesis, peripheral intravenous catheter placement, intra- articular injection, and incision and drain-age. Clinicians can use POCUS to both pinpoint important anatomic structures such as vessels and nerves, and visualize the needle tip during a procedure. This dynamic, or real-time, guidance enables the provider to accurately target and avoid certain structures, thereby decreasing the likelihood of procedural complications.

POCUS guidance allows for the safe and precise placement of peripheral nerve blocks in appropriate patient popula-tions and clinical scenarios1 (Table 1). Additionally, there is evidence that its use may improve the onset, quality, and duration of peripheral nerve blocks compared to non-ultrasound-guided techniques.2-4

CRITICAL DECISIONWhat are the indications and contraindications for placement of an ultrasound-guided peripheral nerve block?

Indications for ultrasound-guided regional anesthesia include pain control when parenteral analgesia may not be desirable. The procedure also can be used for patients undergoing uncomfortable procedures or seeking relief from chronic underlying conditions.

Contraindications to placement include infection overlying the site of needle insertion; the need for serial neurological examinations; an allergy to local anesthetic; co-morbidities that could be exacerbated by motor function blockade of the peripheral nerve; and altered mental status or the presence of sedation, which may prevent the patient from accurately reporting pain or paresthesia during the injection of anesthetic.

exhibits 5 out of 5 strength with plantar and dorsiflexion of the right foot, intact sensation, and excellent distal capillary refill of all toes.

■ CASE THREEAn otherwise healthy 30-year-

old woman presents with a large laceration over her left palm, which she sustained while slicing a bagel with a kitchen knife. The patient is right-hand dominant, and her tetanus vaccination is up to date. The physical examination reveals a 4-cm linear laceration extending from the base of the left third metacarpophalangeal (MCP) joint to the skin overlying the thenar eminence. No tendons or blood vessels are visible.

Her motor and sensory function in the median, radial, and ulnar nerve distributions are fully intact, and there is excellent capillary refill in all fingers. She is able to fully extend and flex at all MCP, proximal interphalangeal, and distal interphalangeal joints.

April 2016 n Volume 30 Number 4 5

CRITICAL DECISIONWhat are the procedure’s most important risks and benefits?

The potential benefits of an ultrasound-guided nerve block include enhanced pain control, a decreased need for systemic analgesia, less nausea, and a reduced time to discharge if sedation is not required to complete the procedure. Accurate and appropriate use of POCUS guidance decreases the risks of possible infection, transient and/or chronic paresthesias, nerve damage, the intravascular injection of anesthetic agent, seizure, cardiac arrest, and failure to achieve adequate anesthesia.

Although rare, another potential complication is local anesthetic systemic toxicity (LAST), which results from the intravascular injection of local anesthetic and can lead to seizures, arrhythmias, and cardiac arrest.5 Therapy for LAST includes airway management, treatment of seizures with benzodiazepines, and administration of 20% IV fat emulsion (1.5 mL/kg initial bolus of 20% fat emulsion followed by a continuous infusion of 0.25 mL/kg/min until cardiac stability is achieved).6 Prior to placing a peripheral nerve block, the emergency provider should ensure that a functioning IV line is in place, the patient is being observed on a telemetry monitor, and 20% IV fat emulsion is available within the practice environment.

CRITICAL DECISIONWhat medications and needles are required for the performance of ultrasound-guided regional anesthesia?

When determining the appropriate anesthetic to use, the emergency provider should take into account the desired length of time for anesthesia (Table 2). For example, a shorter-acting anesthetic should be used during the laceration repair of an extremity, whereas a longer-acting agent should be used for a hip fracture. 2-chloroprocaine — the shortest-acting local anesthetic — is metabolized by ester hydrolysis in the bloodstream and,

therefore, does not require metabolism in the kidneys or liver.

Other short-acting local anesthetics include lidocaine without epinephrine, and mepivacaine. The use of 2% lidocaine (compared to 1%) enables the provider to utilize a smaller volume of medication. Long-acting local agents include ropivacaine and bupivacaine.

Standard cutting syringe needles can be used to administer anesthetic during an ultrasound-guided peripheral nerve block; however, blunt-tip needles (ie, Tuohy needles) are preferred and may decrease the risk of mechanical injury (eg, tears to the nerve). Blunt-tip needles also provide enhanced tactile feedback as the needle tip passes through fascial planes, which are important anatomical landmarks during certain procedures. Standard needles typically are between 22-and 25-gauge in size; the length can vary between 1.5 and 3 inches, depending on the type of block being performed.

CRITICAL DECISIONWhat general principles and techniques should be employed during the procedure?

As with any procedure conducted in the emergency department, the provider should obtain informed patient consent and discuss the potential risks and benefits of an ultrasound-guided peripheral nerve block.

A focused neurological examination should be performed and documented prior to the procedure. The sensory innervations of the peripheral nerves are important to consider (Table 1). Failure to appreciate this anatomy can lead to placement of an effective peripheral nerve block that does not provide anesthesia to the area of pain or injury.

The skin overlying the needle insertion site should be thoroughly cleaned with povidone-iodine or chlorhexidine. A sterile barrier such as a transparent dressing or ultrasound probe cover (the same cover used

TABLE 1. Sensory Innervation of Peripheral Nerves

Note: The indications, contraindications, complications, and degree of difficulty in performing the procedure vary for each individual nerve.

Sensory Innervations Areas of Expected Anesthesia

Interscalene brachial plexus Shoulder, upper arm, elbow

Superficial cervical plexus Anterior and lateral neck, clavicle, periauricular region

Supraclavicular brachial plexus Entire arm distal to the shoulder and hand

Infraclavicular brachial plexus Elbow, forearm, hand

Axillary brachial plexus Forearm, hand

Median nerve Anterior thumb, anterior index finger, anterior long finger, radial and anterior aspect of ring finger, palm lateral to ring finger

Ulnar nerve Ulnar aspect of ring finger, entire little finger, anterior and posterior palm and hand medial to ring finger

Radial nerve Posterior hand lateral to ring finger

Femoral nerve Anterior thigh and knee, portion of medial lower leg

Popliteal nerve Leg below the knee with exception of portion of medial leg and ankle.

Saphenous nerve Medial leg and ankle

Posterior tibial nerve Sole of the foot

Deep peroneal nerve 1st web space of foot

Sural nerve Posterolateral lower leg/ankle and dorsolateral foot

Saphenous nerve Medial lower leg/ankle

Superficial peroneal nerve Dorsum of the foot

Critical Decisions in Emergency Medicine6

during placement of an ultrasound-guided central venous catheter) should be applied to the surface of the linear ultrasound transducer. The surrounding skin should be draped to ensure a sterile technique.

During any ultrasound-guided procedure, optimal positioning improves exposure to the anatomical area and enhances patient comfort. The ultrasound machine should be positioned in such a way that the operator is able to face both the screen and the patient simultaneously, which can help minimize the number of movements needed to perform the procedure.

Next, the targeted nerve should be identified using POCUS. A high-frequency (low-penetration) linear ultrasound transducer (10 to 5 MHz) should be used to identify the location of the nerve and important surrounding structures, including arteries and veins. Peripheral nerves often track parallel and adjacent to vascular structures.

Nerves can be easily identified by their “honeycomb” appearance and increased echogencity (compared to vascular structures); they also exhibit anisotropy, meaning their appearance under ultrasound can vary depending upon the angle of the probe transducer.

Once the target peripheral nerve has

been visualized, the provider should adjust the transducer angle using a subtle rocking motion, which will help determine the probe position that provides optimal visualization of the nerve.

The ApproachThere are two techniques for placing

an ultrasound-guided peripheral nerve block — an in-plane approach, and an out-of-plane approach. “In-plane” (Figures 1 and 2) and “out-of-plane” refer to the relationship of the ultrasound transducer to the needle being used to inject local anesthetic.

The in-plane approach (where the needle enters the skin from the side and directly under and parallel to the probe), compared to an out-of-plane approach (where the needle enters the skin at the center of the ultrasound probe and perpendicular to it), allows for continual ultrasound visualization of the needle tip during insertion and its progression toward the target nerve.

Based on the location and spatial considerations of specific nerve blocks, an out-of-plane approach is occasionally necessary (Figures 3 and 4). It is important to note that when using this technique, the needle tip easily can be mistaken for the shaft — a pitfall that increases the risk of inadvertent damage to surrounding structures and inaccurate placement of local anesthetic.

A “time-out” should be performed to confirm that informed consent has been obtained and verify the identity of the patient necessitating the procedure. Next, a small amount of local anesthetic

FIGURE 1. Top: Orientation of transducer and needle for in-plane approach. Bottom: Appearance of needle tip and needle shaft on POCUS with in-plane approach.

TABLE 2. Unique Properties of Local Anesthetic Agents Note: The smallest dose to achieve anesthesia should be used. The amount will vary depending on the location and type of peripheral nerve block.

Medication Maximum Dose Time of Onset Duration

2-chloroprocaine without epinephrine

11 mg/kg 10-15 minutes 1 hour

1.5% mepivacaine without epinephrine

5 mg/kg 10-20 minutes 2-3 hours

2% lidocaine with epinephrine

4 mg/kg 10-20 minutes 2-5 hours

0.5% bupivacaine with epinephrine

2 mg/kg 15-30 minutes 5-15 hours

Local Anesthetics: Clinical Pharmacology and Rational Selection website. http://www.nysora.com/regional-anesthesia/foundations-of-ra/3492-local-anesthetics-clinical-pharmacology-and-rational-selection.html. Published October 14, 2013. Accessed September 15, 2015.

April 2016 n Volume 30 Number 4 7

should be injected subcutaneously at the planned site of needle insertion. The emergency provider should use his or her dominant hand to hold the needle, and non-dominant hand to firmly anchor the ultrasound probe against the patient’s skin.

The needle should then be inserted using an in-plane approach to provide continual dynamic ultrasound guidance. The needle tip should be slowly advanced toward the target nerve under continuous visualization. Subtle changes in the slope of the needle angle can be made to direct the tip (either more deeply or superficially) toward the target nerve to avoid surrounding structures. Once the tip has been advanced to a position adjacent to the nerve, the provider should aspirate to ensure that the needle is not located in a vessel. If there is no aspiration of blood, a small volume of anesthetic should be injected

into the tissue adjacent to the nerve.There should be minimal resistance

to the injection of anesthetic. Resistance, pain, or paresthesia may indicate incorrect positioning — particularly an intraneural needle tip position, which can lead to nerve damage and should be avoided. If the patient experiences pain or paresthesias, the provider should stop the injection immediately and reposition the needle tip to a location adjacent to the target nerve. The procedure only should be restarted if the previous paresthesias and/or discomfort have been fully resolved.

Anesthetic should be injected adjacent to the nerve to allow the medication to track toward and subsequently surround the nerve sheath. This leads the hyperechoic nerve to become encased by the local anesthetic, which appears as anechoic (black) fluid on ultrasound. Prior to beginning any procedure, serial

examinations should be performed over the next 10 to 15 minutes to assess the efficacy of the nerve block.

CRITICAL DECISIONWhich specific peripheral nerve blocks are of greatest utility in the emergency department, and how should the placement of each type be approached?

While a complete overview of all peripheral nerve blocks is beyond the scope of this review, Table 1 lists those that are most frequently performed in the emergent setting. Here, we focus on three of the safest and most common blocks that can be used when performing ultrasound-guided regional anesthesia.

Fascia Iliaca BlockThe fascia iliaca block is used to

anesthetize structures innervated by the femoral nerve by delivering local anesthetic in the fascial plane lateral to the femoral nerve. The local anesthetic then tracks medially to contact and anesthetize the femoral nerve. Given that the target injection location is not the femoral nerve itself, there is a decreased risk of mechanical nerve injury, intraneural injection of anesthetic, and intravascular injection of anesthetic into the femoral artery or vein.

A fascia iliaca block leads to anesthesia of the anterior and medial thigh down to the knee, as well as a portion of the medial lower leg and ankle. Successful placement of a fascia iliaca block requires identification of several key anatomic structures: the femoral vein, femoral artery, femoral nerve, fascia lata, and fascia iliaca.

Using POCUS, the femoral vein and artery should be visualized in the transverse axis just distal to the inguinal ligament. The femoral nerve, which is located just lateral to these structures, may be difficult to see due to decreased echogenicity; subtle adjustments to the ultrasound transducer should be made to maximize visualization. The provider should then slide the ultrasound transducer laterally while maintaining the transverse axis (Figure 5) until two linear hyperechoic structures are visualized — the fascia lata (superficial

FIGURE 2. In-plane approach appearance of needle tip and shaft as they progress toward target.

Critical Decisions in Emergency Medicine8

hyperechoic linear structure) and fascia iliaca (deeper hyperechoic linear structure that lies just superficial to the iliacus muscle).

The provider can use either an out-of-plane or an in-plane approach to perform this peripheral nerve block; however, an in-plane approach is recommended. The skin overlying the site of needle insertion should be locally anesthetized to decrease patient discomfort. The clinician’s non-dominant hand should be used to hold the ultrasound transducer, while the dominant hand advances the needle toward the fascial layers.

The provider may sense a tactile loss of resistance, or “pop,” as the needle is advanced; two “pops” may be felt as it traverses the fascia lata and fascia iliaca. This tactile sensory feedback is more pronounced with a blunt-tipped needle. Following the second “pop,” the needle tip should be visualized just deep to the fascia iliaca. Aspiration should be performed to confirm an extravascular position, followed by an injection of local anesthetic (1 to 2 mL).

If the needle tip is in the proper location, the anesthetic will be seen on ultrasound deep to the fascia iliaca and spread towards the femoral nerve. The remaining anesthetic should be injected only after the correct needle position has been confirmed. A relatively high volume of local anesthetic (20 to 30 mL) is necessary because the medication must track medially in the fascial plane to come into contact with the femoral nerve. Note that the goal is to avoid injecting the anesthetic directly adjacent to the femoral nerve. Rather, this is a compartment block, which relies on a high volume of medication to track along the fascial plane.

Posterior Tibial Nerve BlockA posterior tibial nerve block

provides excellent anesthesia to a region of the body that is notoriously difficult to infiltrate with a local agent — the heel and sole of the foot.7 First, the patient should be positioned lying prone on the examination bed to facilitate access to the posterior aspect of the lower leg. Next, the emergency provider should use the linear high-

FIGURE 3. Top: Orientation of transducer and needle for out-of-plane approach. Bottom: Appearance of needle tip and shaft on POCUS.

FIGURE 4. Out-of-plane approach appearance of the needle tip as it progresses toward the target.

April 2016 n Volume 30 Number 4 9

frequency ultrasound transducer over the medial aspect of the distal leg (just proximal to the medial malleolus) to visualize the key anatomic structures: the posterior tibial artery, vein, and nerve.

Just adjacent to the posterior tibial artery is the posterior tibial nerve, which can be identified by its round, hyperechoic, “honeycomb” appearance (Figure 6). The skin overlying the site of needle insertion should be locally anesthetized to decrease patient discomfort. The clinician’s non-dominant hand should be used to hold the ultrasound transducer, while the dominant hand holds the needle.

Given the narrow overlying skin space and relatively superficial location of the posterior tibial nerve, an out-of-plane approach may be necessary. Once the needle tip is advanced and visualized adjacent to the nerve, the provider should aspirate to ensure the needle tip is in an extravascular location. Anesthetic (1 to 3 mL) should then be injected into the subcutaneous tissue adjacent to the posterior tibial nerve. The dynamic use of POCUS allows real-time visualization of the nerve sheath bundle as it becomes surrounded by the anesthestic agent, and prevents the intraneural injection of medication directly into the nerve bundle.

Median Nerve BlockSensory distribution of the palm

is provided by both the median and ulnar nerves. The ulnar nerve typically provides sensory function to the little finger, the ulnar aspect of the ring

finger, and the palm from the medial aspect to the area directly proximal to the ulnar aspect of the ring finger. The median nerve typically innervates the remainder of the anterior hand and palm not innervated by the ulnar nerve, including the radial aspect of the ring finger, the entire long and

index fingers, the medial aspect of the thumb, and the area of the palm located directly proximal to the radial aspect of the ring finger and extending laterally to the thumb.

Most emergency physicians are capable of performing peripheral nerve blocks of the radial, median, and ulnar nerves following a brief training intervention.8 The risks of performing a peripheral nerve block at the level of the mid-forearm are less than those associated with more proximal nerve blocks at the level of the brachial plexus; yet, there is evidence to suggest equal efficacy of these locations for the upper extremity.9 Additionally, utilization of the median nerve block also prevents tissue distortion related to the administration of local anesthetic directly into the wound edges.

The patient’s arm should be positioned flat and supinated on the procedure table. The linear high-

FIGURE 5. Top: Orientation of transducer for fascia iliaca block. Bottom: POCUS appearance of femoral artery (A), femoral nerve (dashed circle), fascia lata (stars), and fascia iliaca (arrows).

n An in-plane approach decreases the risk of mistaking the needle shaft for the tip (as opposed to the out-of-plane approach).

n Ensure that the sensory innervation of the targeted peripheral nerve corresponds to the area of the patient’s pain.

n Subtle adjustments to the angle of the transducer probe can significantly alter the visibility of the “honeycomb” appearance of peripheral nerves due to the principle of anisotropy.

n The use of 2% lidocaine (as opposed to 1%) reduces the volume of anesthetic required to attain adequate anesthesia.

Critical Decisions in Emergency Medicine10

frequency ultrasound transducer should be placed over the proximal to mid-forearm to identify and localize the median nerve.10 The median nerve is located between the flexor digitorum superficialis and profundus muscles at the proximal to mid-forearm, and should not track with an adjacent vessel (Figure 7).

If there appears to be a vessel adjacent to the “honeycomb” nerve, the provider should slide the ultrasound probe distally to locate an area where the median nerve is not adjacent to a vessel. Once the optimal area of needle insertion has been determined, the skin should be treated with local anesthetic to decrease patient discomfort during the procedure.

The provider’s non-dominant hand should hold the ultrasound transducer, and the dominant hand should hold the needle. Either an out-of-plane or an in-plane approach can be used to guide the needle tip. Aspiration should then be performed to confirm an extravascular location of the needle tip followed by an injection (approximately 1 to 3 mL) of anesthetic into the subcutaneous tissue adjacent to the median nerve.

Continuous dynamic POCUS guidance provides visualization of the separation of the muscle fascial layers followed by deposition of local anesthetic surrounding the median nerve.

SummaryUltrasound-guided peripheral nerve

blocks are safe and effective adjuvants or alternatives to providing analgesia in the emergency department. For certain patient populations and conditions, the procedure offers potential advantages over systemic analgesia by avoiding medication side effects that can lead to adverse events and increased lengths of stay.

Ultrasound-guided regional anesthesia allows clinicians to visualize and identify key anatomical structures and maintain constant visualization of the needle tip during the procedure. Ensuring appropriate patient selection, using adequate monitoring and ancillary staff, and having 20% IV fat emulsion available in the rare event of LAST are needed for safe performance. Decisions

FIGURE 6. Top: Orientation of transducer for posterior tibial nerve block. Bottom: POCUS appearance of posterior tibial nerve (dashed circle) and posterior tibial artery (arrow).

FIGURE 7. Top: Orientation of transducer for median nerve block. Bottom: POCUS appearance of median nerve (dashed circle).

April 2016 n Volume 30 Number 4 11

CASE RESOLUTIONS■ CASE ONE

The elderly woman with hip pain following a fall was correctly identified as an appropriate candidate for an POCUS-guided fascia iliaca block. (Fascia iliaca blocks appear to provide equal — possibly superior — analgesia compared to systemic agents, and are reasonable options for emergency physicians who have received brief training interventions.)11-14

The patient’s history of chronic lung disease placed her at higher risk for complications associated with hypopnea (eg, respiratory depression associated with the use of oral or systemic opioid medications). Additionally, the use of a fascia iliaca block decreased the need for systemic analgesia, which can elevate the risk of medication-induced delirium in elderly patients.13,15

Following careful documentation of the patient’s neurological examination and an orthopedic consultation, the clinician performed

a POCUS-guided block with bupi-vacaine. Within 20 minutes, the patient’s left hip pain was significantly decreased. She was admitted to the orthopedic service overnight and underwent operative repair of her fracture the following morning.

She required minimal additional analgesia beyond the nerve block prior to her operation, and did not experience the side effects associated with opiate use in elderly patients with chronic lung disease. The appropriate use of a long-acting anesthetic allowed for extended pain relief while the patient awaited operative repair. The strategy also eliminated the need for an additional fascia iliaca block or opiate medications.

■ CASE TWOThe clinician correctly identified

that the young man with a foot laceration would be an appropriate candidate for a POCUS-guided posterior tibial nerve block with a short-acting local anesthetic agent. Within 10 minutes of administration,

a primary repair of the laceration was performed successfully. Because no additional anesthesia or analgesia was required, the patient could be discharged immediately following completion of the laceration repair.

■ CASE THREEThe emergency provider concluded that

the woman with a laceration to her left hand would be an excellent candidate for an POCUS-guided median nerve block with a short-acting local anesthetic agent. Successful placement of the block allowed anesthesia of the palm with a smaller volume of medication than would have been required with the direct infiltration of local anesthetic into the laceration.

Approximately 10 minutes later, the provider performed a primary repair of the laceration without the need for additional anesthetics or opiate analgesia. Precise approximation of skin edges was facilitated by the absence of tissue distortion and edema. The patient was discharged immediately following the procedure.

n Performing a peripheral nerve block on a patient with altered mental status or intoxication. Reliable patient feedback regarding pain and paresthesia during an anesthetic injection is needed to perform the procedure safely.

n Failing to perform and document a thorough examination of the affected bodily region prior to placement of a nerve block.

n Failing to confirm a functional IV line and telemetry monitoring during placement of a peripheral nerve block.

n Failing to ensure the availability of 20% IV fat emulsion within the practice environment. The antidote must be on hand for the rare patient who suffers from local anesthetic systemic toxicity (LAST).

about which local anesthetic to use should be guided by the patient’s injury and the time required for anesthesia.

REFERENCES1. Bhoi S, Sinha TP, Rodha M, et al. Feasibility and safety

of ultrasound-guided nerve block for management of limb injuries by emergency care physicians. J Emerg Trauma Shock. 2012;5(1):28-32.

2. Liu SS. Evidence basis for ultrasound-guided block characteristics onset, quality, and duration. Reg Anesth Pain Med. 2015.

3. Salinas FV. Ultrasound and review of evidence for lower extremity peripheral nerve blocks. Reg Anesth Pain Med. 2010;35(2 Suppl):S16-25.

4. Walker KJ, McGrattan K, Aas-Eng K, Smith AF. Ultrasound guidance for peripheral nerve blockade. Cochrane Database Syst Rev. 2009;(4):CD006459.

5. Neal JM, Bernards CM, Butterworth JF 4th, et al. ASRA practice advisory on local anesthetic systemic toxicity. Reg Anesth Pain Med. 2010;35(2):152-161.

6. Weinberg GL. Treatment of local anesthetic systemic toxicity (LAST). Reg Anesth Pain Med. 2010;35(2):188-193.

7. Redborg KE, Antonakakis JG, Beach ML, et al. Ultrasound improves the success rate of a tibial nerve block at the ankle. Reg Anesth Pain Med. 2009;34(3):256-260.

8. Liebmann O, Price D, Mills C, et al. Feasibility of forearm ultrasonography-guided nerve blocks of the radial, ulnar, and median nerves for hand procedures in the emergency department. Ann Emerg Med. 2006;48(5):558-562.

9. Soberon JR, Bhatt NR, Nossaman BD, et al. Distal peripheral nerve blockade for patients undergoing hand surgery: A pilot study. Hand (N Y). 2015;10(2):197-204.

10. McCartney CJ, Xu D, Constantinescu C, et al. Ultrasound examination of peripheral nerves in the forearm. Reg Anesth Pain Med. 2007;32(5):434-439.

11. Foss NB, Kristensen BB, Bundgaard M, et al. Fascia iliaca compartment blockade for acute pain control in hip fracture patients: A randomized, placebo-controlled trial. Anesthesiology. 2007;106(4):773-778.

12. Godoy Monzón D, Vazquez J, et al. Pain treatment in post-traumatic hip fracture in the elderly: regional block vs. systemic non-steroidal analgesics. Int J Emerg Med. 2010;3(4):321-325.

13. Beaudoin FL, Haran JP, Liebmann O. A comparison of ultrasound-guided three-in-one femoral nerve block versus parenteral opioids alone for analgesia in emergency department patients with hip fractures: a randomized controlled trial. Acad Emerg Med. 2013;20(6):584-591.

14. Haines L, Dickman E, Ayvazyan S, et al. Ultrasound-guided fascia iliaca compartment block for hip fractures in the emergency department. J Emerg Med. 2012;43(4):692-697.

15. Godoy Monzón D, Iserson KV, Vazquez JA. Single fascia iliaca compartment block for post-hip fracture pain relief. J Emerg Med. 2007;32(3):257-262.

Critical Decisions in Emergency Medicine12

IN THE EMERGENCY DEPARTMENT A Picture’s Worth 1,000 Words

THE OFFICIAL CME PUBLICATION OF THE AMERICAN COLLEGE OF EMERGENCY PHYSICIANS

Spring 2016

DIAGNOSTIC IMAGING SPECIAL EDITIONINSIDE THIS ISSUE

Bedside Ultrasonography Diagnosis of Ocular Pathology n Ultrasound-Guided Central Venous Access Ultrasound Use in Resuscitation n Imaging in Blunt Cervical Spine Trauma

Is Coronary CT Angiography Ready for Your Emergency Department?

Inside this issue• Bedside Ultrasonographic Diagnosis of

Ocular Pathology• Ultrasound-Guided Central Venous Access• Ultrasound Use in Resuscitation• Imaging in Blunt Cervical Spine Trauma• Coronary CT Angiography

Earn up to 12.5 AMA PRA Category 1 creditsTM to ful� ll your annual Procedures and Skills CME requirement.

3 EASY WAYS TO ORDER

acep.org/cdem [email protected] 800-798-1822 Ext. 5

SHARPEN YOUR SIGHTS AT THE BEDSIDE with Critical Decision’s Special

Diagnostic Imaging Edition – packed

with case-based intel on the practical

ED applications of ultrasound, CT,

radiography and more.

Which imaging tools should guide

your next workup?

We’ve got the answers.

NewRelease

April 2016 n Volume 30 Number 4 13

Since the introduction of the Haemophilus influenza type b vaccine in 1988, the incidence of H. flu type B meningitis has radically declined. Streptococcus pneumoniae now is the most common pathogen for bacterial meningitis in children outside the neonatal period, despite the develop ment of the 7-valent pneumococcal conjugate vaccine (PCV7). Neisseria meningitides is the second most common pathogen. Although the rate has decreased in older age groups, the incidence of bacterial meningitis has remained steady in infants younger than 2 months.

Classic signs and symptoms of the disease are less common in younger age groups. Older children may have fever, headache, photophobia, seizure, or changes in mental status. Infants, however, can present with less specific signs such as hypothermia, vomiting, poor feeding, or paradoxical irritability. Physicians should evaluate for shock, volume status, neurologic deficits, and signs of increased intracranial pressure. Neck stiffness is an unreliable symptom in children younger than 2 years.

Preferably, cerebral spinal fluid

Bacterial Meningitis Post-PCV7: Declining Incidence and TreatmentKowalsky R, et al. Pediatr Emerg Care. 2013 Jun;29(6):758-766.

The LLSA Literature Review

Critical Decisions in Emergency Medicine’s series of LLSA reviews features articles articles from ABEM’s 2016 Lifelong Learning and Self-Assessment Reading List. Available online at acep.org/llsa and on the ABEM website.

Reviewed by J. Stephen Bohan, MS, MD, FACEP

n S. pneumoniae is still the most common pathogen for bacterial meningitis in children outside the neonatal period.

n In infants younger than 1 month, start ampicillin plus either cefotaxime or an aminoglycoside, and consider acyclovir therapy for herpes simplex virus coverage.

n In children older than 1 month, start vancomycin plus a third-generation cephalosporin.

KEY POINTS

(CSF) laboratory studies and a complete blood count should be acquired prior to the initiation of antibiotics. If the child is hemodynamically unstable or shows signs of increased intracranial pressure requiring computed tomography prior to the performance of a lumbar puncture, antibiotics can be administered after a blood culture has been obtained. No single laboratory test is completely diagnostic, and interpretation is especially difficult in pretreated patients. Notably, most children with CSF pleocytosis are negative for bacterial meningitis.

An enteroviral polymerase chain reaction test also may be helpful in sorting out the cause of the pleocytosis. The bacterial meningitis scoring system can be used in children older than

2 months to identify low-risk patients for whom outpatient management may be appropriate.

Empiric antibiotic therapy with parental antibiotics should be started expeditiously in any child with suspected bacterial meningitis. In infants younger than 1 month, ampicillin plus either cefotaxime or an aminoglycoside should be administered. Acyclovir therapy for human herpesviruses also should be considered, which will cover group B strep, L. monocytogenes, and E. coli. In children older than 1 month, vancomycin plus a third-generation cephalosporin will cover S. pneumoniae and N. meningitides. Steroids are unlikely to be beneficial and generally are not recommended.

By Marija M. Lum, MD University of Utah School of Medicine, Emergency Medicine Residency, Salt Lake City, Utah

Critical Decisions in Emergency Medicine14

The use of a Sengstaken-Blakemore tube usually is reserved for

patients with acute variceal bleeding who have failed to respond

to all other standard emergency therapies. Though rarely

employed, this balloon tamponade device can offer lifesaving

support for high-risk patients with uncontrolled hemorrhage.

The Critical Procedure

CONTRAINDICATIONSn There are few contraindications, given that this procedure marks a final attempt to manage hemorrhage

and prevent death. The placement of a balloon tamponade device should be avoided, however, in patients predisposed to esophageal rupture (eg, esophageal stricture or recent esophageal/gastric surgery).

Risks and BenefitsIn general, the success rate of balloon

tamponade placement ranges from 60% to 90%. The most obvious benefit is hemorrhage control, and the ability to get the patient to definitive treatment of the varices. There are many risks associated with Sengstaken-Blakemore tubes, and complications occur in up to 14% of patients. The most common risk is the inability to control bleeding, resulting in death. Airway complications also are possible — the most concerning of which is obstruction, thus emphasizing the need for early intubation. Aspiration pneumonitis from placement in the non-intubated patient also is a risk.

SENGSTAKEN-BLAKEMORE TUBE PLACEMENT

By Christopher Sampson, MD, FACEPDr. Sampson is the associate program director and an assistant professor at the University of Missouri in Columbia, Missouri.

gastric balloon

length marks

esophageal aspiration opening

esophageal balloon

gastric balloon port

gastric aspiration port

esophageal balloon port

esophageal aspiration port

gastric aspiration opening Û This double-balloon tamponade system has undergone relatively few changes since its initial development 65 years ago. Its three major components are the gastric balloon, esophageal balloon, and gastric suction port.

Reviewed by Steven J. Warrington, MD

April 2016 n Volume 30 Number 4 15

TECHNIQUE1. Notify staff and intubate the

patient (ideally, prior to starting the procedure).

2. Test the device balloons prior to use to ensure no leaks are present.

3. Secure a standard nasogastric tube with silk sutures (approximately 3 cm proximal to esophageal balloon) to create an esophageal aspiration. Or, alternatively, insert a nasogastric tube after placement of the Sengstaken-Blakemore tube.

4. Insert the tube through the patient’s mouth (as would be done during placement of a nasogastric tube), and stop when the 50-cm mark has been reached.

5. Apply continuous suction to the esophageal and gastric ports.

6. Confirm placement with a syringe and the injection of air while auscultating over the stomach. Once confirmed, inflate the gastric portion with 50 mL of air.

7. Obtain a chest x-ray to confirm balloon placement in the stomach (ultrasound may be considered).

8. Connect a manometer via Y tube and inflate with an additional 200 mL of air (for a total of 250 mL). Measure pressures every 100 mL. High pressure suggests esophageal displacement (ie, a 15-mmHg increase); in such cases, consider deflation and a reattempt.

The procedure also poses a risk of esophageal or gastric perforation. Ulceration from the tube or balloon can occur in the oral cavity, esophageal, and gastric mucosa.

Reducing Side EffectsAny patient undergoing this

procedure ideally should be intubated to control the airway and decrease potential complications such as obstruction or aspiration. It is important to not overinflate the esophageal or gastric balloons, as this

can increase the risk of ulceration.

Personal protective equipment

should be used to reduce the risk of

exposure to blood. Additionally, the

use of radiography after initial minimal

inflation, along with the measurement

of pressures, may decrease the danger

of esophageal perforation.

AlternativesEmergency endoscopy, if available,

is the preferred treatment for direct

hemorrhage control.

Special ConsiderationsAdequate analgesia and sedation

must be ensured following tube insertion. It is important to note that placement of a Sengstaken-Blakemore tube is only a temporizing procedure; arrangements should be made for more definitive care.

Bedside ultrasound can be used successfully to confirm placement of the gastric balloon. The benefit of ultrasound is rapid bedside confirmation, when compared with the time to obtain a chest radiograph.

9. Close the gastric port with clamps (if available) once the device has been inflated. Do not use bare hemostats. If no clamps are available, cover the surface with red rubber tubing or tape prior to placement.

10. Apply traction (~1kg) using a roller bandage and a 1-liter bag of fluids hung over an IV pole. Mark the depth of tube at the mouth. (The tube will stretch as it warms from body heat.)

11. Secure the tube with an endotracheal tube attachment device.

12. Suction the lavage port and nasogastric tube. If bleeding continues, inflate the esophageal balloon (ideally to 30 mmHg). Use the manometer and Y tube to avoid over inflation. If hemorrhage persists, inflate the esophageal balloon to 45 mmHg.

13. Clamp the esophageal tube (as in step 9).

Chest x-ray demonstrates a Sengstaken-Blakemore tube in situ, as well as a left central venous catheter and an endotracheal tube.

Critical Decisions in Emergency Medicine16

A 68-year-old man with chest pain and weakness.

The Critical ECGST, rate 105, infero-antero-lateral ischemia, consider left main coronary artery (LMCA) occlusion.

Diffuse ischemia is diagnosed based on the ST-segment

depression in multiple leads. Of note, however, is the

From Mattu A, Brady W, ECGs for the Emergency Physician 2. London: BMJ Publishing; 2008:11,23. Reprinted with permission.

By Amal Mattu, MD, FACEPDr. Mattu is a professor, vice chair, and director of the Emergency Cardiology Fellowship in the Department of Emergency Medicine at the University of Maryland School of Medicine in Baltimore.

ST-segment elevation (STE) in lead aVR. In the presence of acute cardiac ischemia, STE in lead aVR is

strongly suggestive of occlusion of the left main coronary artery (LMCA). When the magnitude of the STE

in aVR is greater than the STE in lead V1 (as is seen here), or when there is simultaneous STE in leads aVR

and aVL, the specificity for LMCA occlusion increases.1,2 Involvement of the LMCA in ACS is associated

with 70% mortality without prompt invasive therapy (percutaneous intervention, bypass surgery). Medical

management alone is ineffective.

This patient immediately was sent for catheterization and was found to have a 95% occlusion of the LMCA.

REFERENCES1. Kurisu S, Inoue I, Kawagoe T, et al. Electrocardiographic features in patients with acute myocardial infarction associated with left main coronary artery occlusion.

Heart. 2004;90:1059-1060.

2. Yamaji H, Iwasaki K, Kusachi S, et al. Prediction of acute left main coronary artery obstruction by 12-lead electrocardiography. J Am Coll Cardiol. 2001;38:1348-1354.

CORRECTION. We regret that the ECG rhythm strip in the March 2016 issue was published in error. Although it has been corrected in the online version of the Critical ECG, the image in the print edition does not reflect the Type I Wellens T waves indicated in the case presentation.

April 2016 n Volume 30 Number 4 17

FROM THE EM MODEL17.0 Toxicologic Disorders

17.1 Drug and Chemical Classes

17.1.10 Antipsychotics

LESSON 8

By Jonathan Glauser, MBA, MD, FACEP; and David Peters, MDDr. Glauser is a professor of emergency medicine and Dr. Peters is a resident in the Department of Emergency Medicine at Case Western Reserve University School of Medicine in Cleveland, Ohio.

Reviewed by Sharon Mace, MD, FACEP

n What medications are available to the emergency physician for control of the acutely agitated patient?

n Which agents demonstrate lethal or “can’t-miss” complications?

n What is the clinical significance of QTc interval prolongation, and which agents are most likely to cause it?

n What is the link between antipsychotic drugs and sudden cardiac death?

n How should prolongation of the QT interval be managed, and what is the optimal therapy for AAP-induced torsades de pointes?

n Which agents place patients most at risk for metabolic syndrome?

n How should atypical antipsychotic overdose be managed, and what cardiovascular effects should be anticipated?

n What specific characteristics and complications are associated with common atypical antipsychotics?

CRITICAL DECISIONSOBJECTIVESOn completion of this lesson, you should be able to:

1. Describe the advantages that justify the use of newer antipsychotics

2. Identify the specific side effects associated with individual agents used in the emergency management of the psychiatric patient.

3. List the most effective pharmacologic options for the management of the agitated patient.

4. Define the risks for QT interval prolongation posed by respective second-generation antipsychotic agents.

5. Indicate appropriate instances in which individual antipsychotic agents may be contraindicated in the emergency setting.

6. Discuss the distinguishing characteristics and indications of individual antipsychotic drugs.

7. Recognize specific indications for the use of second-generation antipsychotic agents in the emergency department.

Second-Generation Antipsychotics

Mindful Medicine

Critical Decisions in Emergency Medicine18

CASE PRESENTATIONS

■ CASE ONEA 45-year-old man presents with

fatigue, shortness of breath, and bilateral leg swelling over the past 48 hours. He reports that the fatigue began slowly, but has progressed to the point that he is unable to walk a single block or up a flight of stairs without becoming extremely tired and out of breath. He says the leg swelling started at about the same time, and has been getting progressively worse. He denies a rash, pain in his legs or chest, recent illness, immobilization, recent travel, or surgery.

The patient’s medical history is significant for hypertension, which is controlled by an oral agent; hyper-lipidemia, and schizophrenia. He reports taking hydrochlorothiazide and atorvastatin, and notes undergoing recent changes to his psychiatric medication regimen; however, is unsure of the specific drug names. Vital signs are heart rate 104, blood pressure 108/68, respiratory rate 24, temperature 38.8◦C (100.1◦F), and oxygen saturation 93% on room air.

The patient is sitting upright in bed; he is using accessory muscles to breathe, but is able to talk in nearly complete sentences. Jugular venous distension is present on the patient’s neck. No heart murmur is detected on the cardiac examination. The

lungs show bilateral trace crackles. Bilateral pitting edema (3+) is noted on the patient’s legs; however, they are nontender to palpation. No rashes are observed. An ECG is performed, which reveals lateral T-wave inversions with short runs of premature ventricular complexes.

■ CASE TWOA 27-year-old woman is brought to

the emergency department by family members, who are concerned about an ingestion of medications that occurred approximately 90 minutes prior to arrival. The patient has been taking medications for depression, including fluoxetine, amitriptyline, and risperidal; and also has been undergoing treatment for bronchitis.

On arrival, it is unclear which drugs she may have taken and in what quantity; however, she admits attempting suicide in response to a breakup with her boyfriend.

On examination, she is drowsy, and is slow to respond to questions. Vital signs are heart rate 112, respiratory rate 14, blood pressure 124/76, and temperature 37.9◦C (100.3◦F). Her pupils are 2 mm and symmetrical; and her skin is dry, with needle marks visible on her arms and neck. Tachycardia is noted, which improves with the administration of a normal saline bolus.

A psychiatric consultation is

requested, after which the patient loses consciousness and a pulse. The cardiac monitor reveals a wide-complex tachycardia. Intravenous magnesium is administered, and the patient is cardioverted successfully.

■ CASE THREEA 36-year-old man presents with

psychosis. He reports hearing voices through his implanted tooth that are telling him to kill an ex-boyfriend of an ex-girlfriend to prevent the earth from being “wiped out by a meteor.” Historically, his behavior has been difficult to control with antipsychotic medications. The one agent he has been taking for the past 8 months seems to have controlled his bizarre thoughts; however, the “voices” returned recently when he ran out of the medication. The patient’s paranoia is further evidenced by his screaming rants about Elvis “delivering the world from Buddhist monk terrorists.”

The emergency physician orders a toxicology screen and requests a bed in the psychiatric unit. Although it is difficult to get near the patient to obtain vital signs, his pulse is 105 at rest, and blood pressure is 144; his respiratory rate is harder to ascertain. The toxicology screen, including blood alcohol measurements, is negative for all substances tested. The psychiatry consultant prepares to transfer the patient to an outside facility — any outside facility.

Second-generation or atypical antipsychotics (AAPs) were designed in response to the undesirable side effect profiles of their first-generation counterparts. The efficacy and reduced risks of these new medications has led to their growing popularity in the emergency department for chemical restraint and the treatment of a broad range of mental disorders. It is incumbent upon clinicians to be familiar with the efficacy, optimal use, and untoward effects of these increasingly common agents, and understand how to detect and manage AAP overdose.

A Brief HistoryIn the 1950s, chlorpromazine

was noted for its role in helping schizophrenics regain control of many of their most troubling symptoms, including paranoia, delusions, and frank psychosis. During the subsequent quarter-century, as many as 40 new “typical” antipsychotic medications were introduced.

Many of these agents posed an infamously high risk for the development of anticholinergic and adrenergic effects, as well as akathisia,

Parkinsonian symptoms, and dystonia, which together constitute extrapyramidal symptoms (EPS). Rare life-threatening conditions such as neuroleptic malignant syndrome also were noted.

Two large subclasses of medications were included in this new breed of drugs: low-potency agents, which resulted in fewer EPS effects, but had more anticholinergic and adrenergic blocking properties; and high-potency drugs, which carried a higher risk of EPS, but resulted in fewer anticholinergic/adrenergic symptoms. Neither group was able to achieve a

April 2016 n Volume 30 Number 4 19

significant reduction in the overall rate of side effects.

The older agents also were fraught with other significant risks. Thioridazine, which has since been removed from the market, was associated with sudden death and cardiac toxicity in overdose amounts.2-4 Haloperidol was linked to QT prolongation and torsades de pointes (TdP).5

Another downside to typical agents was that none addressed the often-overlooked “negative” symptoms of schizophrenia such as depression, lack of emotion, and inability to experience pleasure or form relationships. For patients and their families, these are among the disease’s most distressing symptoms. For approximately 15 years, patients with schizophrenia had to choose between the side effects of first-generation antipsychotic drugs and the crippling burden of the diagnosis.

The advent of second-generation antipsychotic agents (Table 1) was prompted by the need for more potent agents with comparatively milder side effect profiles than the typical antipsychotics already on the market. Clozapine — the first such drug to be developed — showed initial promise for its effectiveness and reduction in EPS side effects; however, the unfortunate risk of agranulocytosis prevented its widespread use.

During the 1990s, several new medications were discovered, which were as efficacious, or even better than, their first-generation counterparts. These agents offered a decreased risk of EPS, while avoiding the extreme dangers of agranulocytosis. Most second-generation antipsychotics also have a salutary effect on the negative symptoms of schizophrenia, likely due to their antagonist effect on serotonin receptors in the brain.

AAPs in the EDWhile schizophrenia was the initial

target of atypical antipsychotic agents and remains their primary indication, a large number of patients with other psychiatric conditions have benefited from treatment with these drugs in recent years.

AAPs have been used both as monotherapy and as an adjunct in the treatment of depression without psychotic features. While not yet considered a first-line agent, quetiapine — in particular — has shown success as a singular therapy for unipolar depression.6 Adjunctive therapy has grown increasingly common, as well — its popularity partially explained by the sometimes poor response to selective serotonin receptor inhibitor (SSRI) monotherapy. Quetiapine and aripiprazole are the agents most frequently used for this indication.7

Quetiapine monotherapy is the preferred treatment for bipolar disorder; olanzapine and aripiprazole also are popular second- or third-line treatments.8-10 While the first-line therapy for obsessive compulsive disorder (OCD) remains SSRI monotherapy, patients who experience an incomplete response may benefit from the addition of risperidone or aripiprazole.11,12

The many facets of autism spectrum disorder require multiple medications to treat the variety of possible patient presentations. In particular, risperidone, aripiprazole, and olanzapine can be helpful in decreasing the frequency and severity of maladaptive behaviors. Risperidone commonly is used for these symptoms, and also has been effective for mitigating the repetitive behaviors and rigidity sometimes seen with the diagnosis.13-15

CRITICAL DECISIONWhat medications are available to the emergency physician for control of the acutely agitated patient?

Approximately 6% of emergency department visits in the United States are the result of behavioral emergencies caused by psychosis, mania, depression, bipolar disorder, or substance abuse.16 Some of these visits are complicated by destructiveness, disorganization, or dysphoria — a spectrum commonly referred to as “agitation.” This behavior may escalate into situations that can cause harm to the patient or medical staff.

While many of these situations can be diffused with verbal or behavioral de-

escalation techniques, pharmacologic treatments (Tables 3 and 4) sometimes are required for severe acute agitation. The effectiveness of haloperidol, which was established long ago through a multitude of studies, is the antipsychotic drug most widely documented for the management of acute agitation.17

As noted previously, however, first-generation antipsychotics pose a greater risk of extrapyramidal symptoms than AAPs. Benzodiazepines, which also can reduce agitation, commonly are used in conjunction with haloperidol; however, they can increase blood pressure and decrease respiratory drive. Of note, a reduction of the seizure threshold brought about by AAPs can be problematic in agitated patients taking benzodiazepines and suffering from alcohol or sedative withdrawal.18

Short-acting intramuscular AAP formulations (eg, aripiprazole, olanzapine, and ziprasidone) are well suited and best studied for treating these patients. All show a lower propensity for EPS in this setting than older antipsychotics.18 Ziprasidone (10 to 20 mg IM) appears to be particularly efficacious and well tolerated, and may decrease agitation much like haloperidol with lorazepam.19-21

Olanzapine appears to be equivalent to haloperidol, but is faster at reducing agitation with a decreased incidence of dystonia.22 At a dose of 10 mg IM, it has been shown to be superior to lorazepam 2 mg IM, a benefit that is especially pronounced in bipolar patients and demented patients.23 In FDA clinical trials, 11% of patients taking the drug experienced a 20-mmHg drop in systolic blood pressure; it should be avoided in the hypotensive patient.

Aripiprazole was approved by the FDA for agitation in 2006. The usual recommended dose is 9.75 mg, which has been shown to be non-inferior to 6.75 mg of haloperidol IM.18 Oral risperidone (2 mg) appears to be as effective as haloperidol and lorazepam in patients accepting oral medications, and there is no difference in time to reduction of agitation. Risperidone, olanzapine, aripiprazole, and asenapine also are available as oral-

Critical Decisions in Emergency Medicine20

disintegrating tablets; however, these formulations have not been studied for the treatment of agitation.

A 2006 ACEP clinical policy statement on the management of acutely agitated patients recommends atypical antipsychotic monotherapy as a first-line treatment, with comparable evidence for typical antipsychotics alone or in conjunction with benzodiazepines.24

CRITICAL DECISIONWhich agents demonstrate lethal or “can’t-miss” complications?

While atypical antipsychotics may be safer and produce fewer side effects than typical antipsychotics, there are several adverse reactions that can be catastrophic if missed. Several of these medications pose risks associated with QTc interval prolongation, sedation, and anticholinergic effects; and many have unique drug-specific side effects.

Agranulocytosis Agranulocytosis, which is defined by

a granulocyte count of less than 0.5 x 109 and a white blood cell count of less than 1.5 x 109, has been observed with the use of clozapine. The medication is more effective than other antipsychotics

for decreasing the risk of suicide, but the dangers of this potential complication have tempered its use.24,25

The incidence of agranulocytosis per treatment year is approximately 1%. While this statistic might seem insignificant, the mortality risk in such patients may be as high as 35%.27 Half of these deaths occur within 12 weeks of drug initiation, and 75% occur within 6 months; the mechanism of agranulocytosis is unknown.

Case reports indicate that this side effect can occur up to 5 years after initiation of the medication. Weekly monitoring of complete blood counts to detect early drops in white blood cells is standard care for patients taking clozapine. Drug-induced agranulocytosis should be treated by discontinuing the offending agent and aggressively addressing any underlying infection.

MyocarditisMyocarditis is another rare but

potentially life-threatening side effect of antipsychotic drugs. By far the most common offender is clozapine, although the first-generation drug chlorpromazine also has been implicated. Between .015% and .188%

of clozapine users may experience this type of heart inflammation, although retrospective studies have cited mild forms of the disease in as many as 66% of patients.29 Approximately 80% of cases are diagnosed within 4 weeks of drug initiation, and 90% occur before 8 weeks.

Although the pathophysiology of this complication is not definitely known, eosinophilia is common — a response that may suggest an IgE-mediated acute hypersensitivity.30 Clozapine also may spur an elevation in catecholamine release — a factor that might play a role in the development of myocarditis. Mortality approaches 50% in those with the fulminant form of the disease.31

Patients typically present with fever, tachycardia, and chest pain; ECG changes, elevated cardiac markers, eosinophilia, and flu-like symptoms also may be present. The diagnosis is easy to overlook, as its symptoms mirror those seen with the normal titration of clozapine. Patient management should begin with the discontinuation of the offending agent and routine treatment for myocarditis, including the initiation of beta-blockers, ACE inhibitors, and diuretics as indicated.32

TABLE 1. Characteristics of AAPs

Drug Common Dosages

CNS Depression

in ODQTc

Prolongation AntimuscarinicMetabolic Syndrome Unique Features

Clozapine Slow titration to 150-225 mg, 2x/day PO (maintenance therapy only)

+++ + +++ +++ Agranulocytosis, myocarditis, seizures

Risperidone 2-8 mg PO daily for maintenance; 4 mg PO daily for agitation

+ + +/- ++ Dystonic features

Olanzapine 5-20 mg PO daily for maintenance; 5-20 mg PO or IM for agitation

++ + ++ +++ Elevated CK, waxing/ waning level of consciousness

Quetiapine 300-450 mg PO daily +++ + ++ ++ TachycardiaZiprasidone 20-40 PO 2x/day for maintenance;

10 mg IM q2 hrs or 20 mg q4 hrs for acute agitation

++ ++ - + Severity of QTc prolongation

Aripiprazole 10-30 mg PO daily ++ - - + Sedation; relatively safe in overdose

Lurasidone 40-160 mg PO daily ++ - - +/-Paliperidone 6-12 mg PO maintenance,

234 mg IM initial dose followed by 156 mg; 1 week later followed by 39-234 mg IM monthly

+ + + +++ Long-acting preparation

Illoperidone 6-12 mg PO q12 hrs + ++ + ++ Severity of QTc interval prolongation

April 2016 n Volume 30 Number 4 21

Neuroleptic Malignant Syndrome

Neuroleptic malignant syndrome (NMS) is another rare complication of antipsychotic drug use. Again, clozapine accounts for the preponderance of these cases; however, risperidone, olanzapine, quetiapine, ziprasidone, and aripiprazole also have been implicated.33 The incidence of NMS in AAS users is .01 to .02%; mortality is seen in 10% to 30% of cases.

Patients typically develop the condition within hours or days of starting the offending agent, and nearly all presentations are seen within 30 days. The proposed mechanism appears to be related to dopamine-receptor blockage. The primary risk factors include the initiation of a dopamine-blocking drug; use of multiple agents; pregnancy; and, potentially, genetic predisposition.

The classic presentation of drug-induced NMS is a triad of fever, muscle rigidity, and altered mental status. The Diagnostic and Statistical Manual of Mental Disorders defines NMS as exposure with severe muscle rigidity and elevated temperature, as well as two or more of the following symptoms: diaphoresis, dysphagia, tremor, incontinence, changes in level of consciousness, mutism, tachycardia,

elevated or labile blood pressure, leukocytosis or elevated creatine phosphokinase. The most critical aspects of treatment entail the withdrawal of the offending agent, aggressive fluid administration, cooling, and electrolyte correction.

Dantrolene does not appear to benefit overall morbidity or mortality; however, it may decrease symptom duration. It can be used in the critically ill, and combined with the dopamine-antagonists bromocriptine and amantadine; levadopa and apomorphine also can be used. Benzodiazepines are the first-line treatment for controlling any agitation that may develop.27, 33

SeizuresSeizures have been reported in

patients taking several different atypical antipsychotic agents, and all drugs in this class carry a standard warning about this potential side effect. Outside of overdose, it is unclear if these medications can cause seizures on their own; they are likelier to simply lower the threshold in those already susceptible. Clozapine carries the highest dose-related risk of this complication (3%); care should be taken in patients with known seizure disorders.34 Risperidone also poses a slightly higher risk than other AAPs.

CRITICAL DECISIONWhat is the clinical significance of QTc interval prolongation, and which agents are most likely to cause it?

QTc interval prolongation is a notorious and well-studied side effect of atypical antipsychotics (Table 2). Although the exact pathophysiology of the complication is unclear, the leading theory implicates the inhibition of the delayed potassium-rectifier channel, which results in a prolongation of ventricular repolarization.

QT prolongation also appears to contribute to the increase in TdP and sudden death in patients taking AAPs; QTc intervals greater than 500 ms significantly increase this risk. Elderly and female patients and those with concomitant hypokalemia and underlying cardiac disease also are at higher risk. While rare, the risk of torsades de pointes is even greater in those with congenitally-acquired QTc prolongation.35 More than 10 different types of congenital long QT syndromes have been identified.36

At therapeutic doses, iloperidone and ziprasidone are somewhat likelier to cause this complication than other atypical antipsychotics. Quetiapine and clozapine are associated with a moderate

TABLE 2. Medications That Prolong QT Intervals

AAPs Common ED Medications Risk Factors• Clozapine• Risperzole• Ziprasidone• Paliperidone• Illoperidone• Aripiprazole• Olanzipine• Quetiapine

ANTIARRYTHMICS• Procainamide• Dofetilidie, SotalolTYPICAL ANTIPSYCHOTICS• Haloperidol, Droperidol• Thioridazine, ChlorpromazineANTIDEPRESSANTS• Amitriptyline, Desipramine, IMapramine, Doxepin,• FluoxetinANTIBIOTICS• Azithromycin, Clarithromycin, Erythromycin• Levofloxacin, moxifloxacin• Quinine• Pentamidine• Fluconazone, ketonazole, itraconazoleANALGESICS• MethadoneANTIEMETIC• Ondansetron

• Female Sex• Elderly• Multiple QT Prolonging agents• Hypokalemia• Hypomagnesemia• Hypocalcemia• Diuretic use• Hepatic and renal dysfunction• Bradyardia• Congenital long QT syndrome• Heart Failure• Left ventricular hypertrophy• Myocardial infaction• Rapid infusion of QT-prolonging agent• Increased dosage of QT-prolonging agent• Digitalis

Critical Decisions in Emergency Medicine22

risk of QTc interval prolongation; olanzapine and risperidone appear to carry a milder risk. Aripiprazole and lurasidone pose the lowest risk.37

One recent study of 2,356 antipsychotic overdose presentations from 1987 through 2013 analyzed 680 single ingestions and 186 co-ingestions with non-QT prolonging medications. Interestingly, only 1 of the 682 patients who overdosed on thioridazine developed TdP. Abnormal QT intervals developed in 12%, 12%, 5% and 3% of clozapine, risperidone, quetiapine, and olanzapine overdoses, respectively.4

Besides a prolongation in QT or QTc intervals, dispersion (defined as the difference between the shortest and longest QT interval) also may point to the propensity of these drugs to cause TdP.38 Increased variability appears to heighten the risk of TdP, which occurs most often in patient with a heart rate of less than 100 beats per minute.39

Agents such as quetiapine, risperidone, clozapine, and olanzapine are associated with tachycardia (perhaps related to α-1-adrenoreceptor or muscarinic blockades) and may be protective against TdP in cases of overdose.4 It is worth noting that myocardial concentrations (the site responsible for QT prolongation) relative to the plasma concentrations of risperidone, olanzapine, and clozapine are much lower than those of haloperidol, which is associated with QT prolongation and torsades de pointes.5,40

QRS WideningQRS widening is yet another ECG

abnormality seen with the use of AAPs; and QRS prolongation has resulted in a small number of quetiapine overdose presentations. In one review, QRS widening was reported in 10 out of 749 patients with overdose. While 2 of these cases occurred during a TdP rhythm, 186 patients experienced no QRS interval.41

CRITICAL DECISIONWhat is the link between antipsychotic drugs and sudden cardiac death?

Schizophrenic patients taking antipsychotic medications are at an increased risk of sudden cardiac death (SCD). In the 1990s, the link between these agents and QT prolongation, torsades de pointes, and SCD was clearly established. A number of first- and second-generation drugs have been temporarily or permanently pulled from the market due to their effects on the QT interval and significant risk for TdP. Several second-generation antipsychotics (ie, olanzapine, aripiprazole, risperidone, and quetiapine) carry an FDA warning regarding use in elderly patients.42

One study noted a twofold increase in the risk of hospitalization for cardiac arrhythmias or cardiac arrest with the use of several typical agents. There appears to be no association between olanzapine, clozapine, risperidone, and quetiapine and TdP or SCD.43,44 However, an approximate twofold increase in SCD has been linked to typical agents.

The combined Clinical Antipsychotic Trials of Intervention Effectiveness (CATIE) study noted no increase in sudden cardiac death with olanzapine, risperidone, or quetiapine. The results were reported following the discontinuation of the typical agent perphenazine45

CRITICAL DECISIONHow should prolongation of the QT interval be managed, and what is the optimal therapy for AAP-induced torsades de pointes?

In the setting of QTc interval prolongation spurred by an antipsychotic drug (either in normal or overdose concentrations), the first step in patient management is removal of the offending agent. If time permits, a full workup should be initiated, including efforts to rule out structural heart disease. Any electrolyte disturbances should be corrected aggressively.

An admission to the telemetry unit is appropriate if the QTc interval is greater than 500; there is an increase of 60 ms from the patient’s known baseline; signs of ventricular ectopy, T-wave alternans, AV block, or QRS widening; or palpitations or syncope. A prophylactic dose of IV magnesium sulfate should be considered.

If TdP exists and the patient is hemodynamically unstable, prompt nonsynchronized electrical cardioversion should be the first step in management. There are several options available for patients who are conscious and hemodynamically stable. Intravenous magnesium is the preferred treatment. (Two grams given over 1 to 2 minutes aborted 9 of 12 TdP cases in one retrospective study.) A repeat bolus may be considered after 5 to 15 minutes. If the QTc interval remains prolonged, a magnesium sulfate infusion (3 to 20 mg/min) can be initiated.46

An attempt at transvenous overdrive pacing is a reasonable next step. Pacing at a rate above 100 can help shorten the QT interval and abort/prevent torsades de pointes; this is a particularly good option in the bradycardic patient.47 In a study of 9 patients (including those taking quinidine and phenylamine) with mixed causes of prolonged QTc intervals and one congenitally prolonged QT interval, transvenous overdrive pacing successfully aborted all ventricular tachycardia/fibrillation associated with torsades de pointes.48

An isoproterenol bolus and infusion (titrating to a heart rate of 100 beats per minute) also may be beneficial if the patient fails to respond to magnesium. In one small study, 5 of 7 patients started on isoproterenol converted to sinus rhythm.49

There is limited data regarding the effectiveness of class Ib antiarrhythmic agents such as lidocaine in the treatment of TdP. These agents appear to be less predictable than isoproterenol, but have shown benefit in several limited studies.50

IV potassium has been used to successfully convert TdP in a small number of patients with heart failure and quinidine-related QT prolongation — even those with normal serum potassium

TABLE 3. AAP Usage for Agitation

Olanzapine 10 mg IM

Ziprasidone 10-20 mg IM

Aripiprazole 9.75 mg IM

Risperidone 2 mg PO

April 2016 n Volume 30 Number 4 23

concentrations.51 The suggested dose is 0.5 mEq/kg, up to a maximum of 40 mg.36

The clinical significance of prolonged QT intervals in otherwise-healthy patients has been questioned. The concern has been that further prolon-ga tion of the QTc interval (>440 ms) in cases of significant QTc interval prolongation (>500 ms) might predispose patients to TdP or sudden death.

A review of pediatric patients taking risperidone, aripiprazole, and ziprasidone could link no deaths or ventricular dysrhythmias to prolonged QTc intervals. However, tachycardia was observed in the majority of adult patients taking quetiapine or olanzapine; and prolonged QTc intervals were reported in 38 cases. Only 1 patient developed torsades de pointes — a 30-year-old woman who ingested both ziprasidone and amantadine, a medication associated with TdP.

No clinically significant dysrhythmias were clearly attributable to atypical antipsychotic medication toxicity.52 Therefore, while AAPs have been associated with prolonged QT intervals, the clinical significance of its effect in inducing TdP has not been conclusively shown.

CRITICAL DECISIONWhich agents place patients most at risk for metabolic syndrome?

Metabolic syndrome is distinguished by weight gain, diabetes (DM), and dyslipidemia. These factors cause a marked increase in cardiovascular events and other complications of DM. Although nearly every AAP is associated with weight gain, olanzapine and clozapine carry the greatest risk of this side effect; ariprazole, lurasidone and ziprasidone appear to be the most neutral.53-55 While weight gain can be appreciable with these agents, the actual attributable risk is small.53,55

Behavioral, genetic, and pharmacologic components all appear to contribute to the elevated risk of metabolic syndrome. Although the mechanism of action is not entirely known, this side effect may be related to both an increase in appetite and neurohormonal derangements that

escalate the generation of adipose tissue and impair glucose metabolism.56

CRITICAL DECISIONHow should atypical antipsy chotic overdose be managed, and what cardiovascular effects should be anticipated?

Overdose with these agents is a relatively common occurrence; in 2009 alone, more than 43,000 calls were placed to US poison control centers for AAP toxicity.1 These numbers are expected to increase — commensurate with the ever-growing spectrum of conditions for which AAPs are being prescribed.57 As a group, atypical antipsychotic agents elicit a wide variety of presentations; observed effects vary based upon the amount ingested, the health and physiology of the patient, and the drug type.

Central nervous system complica-tions, including sedation, uniformly are seen in cases of AAP overdose. Although NMS is rare, it most commonly is associated with clozapine. Antimuscarinic toxidrome frequently is seen with clozapine, olanzapine,

and quetiapine; it is less common with risperidone, ziprasidone and aripiprazole.57 As with anticholinergic toxidromes, hyperthermia, tachycardia, blurred vision, flushed dry skin, urinary retention, agitation, and hallucinations may be observed.

Cardiac effects (chiefly hypotension) also can be expected. QTc prolongation with progression to TdP and other dysrhythmias are common; however, these complications are dose-related and heavily dependent on the particular agent of abuse.58

Other features unique to certain atypical antipsychotic drugs include dystonic reactions and delayed respiratory depression, which are among the most distinctive side effects of risperidone.59,60 Olanzapine has been noted to produce a mental status that fluctuates between sedation and agitation; it also may cause elevations in creatine phosphokinase (CPK). 61

Treatment should focus on stabilization, the elimination of the offending agent, correction of any laboratory abnormalities, and detection of coingestants. Patients should be

TABLE 4. Agitated Behavior Scale

Rank each category on a 1-4 scale (shown below)1 = absent: the behavior is not present.2 = present to a slight degree: the behavior is present but does not prevent the conduct of other, contextually appropriate behavior. (The individual may redirect spontaneously, or the continuation of the agitated behavior does not disrupt appropriate behavior.)3 = present to a moderate degree: the individual needs to be redirected from an agitated to an appropriate behavior, but benefits from such cueing.4 = present to an extreme degree: the individual is not able to engage in appropriate behavior due to the interference of the agitated behavior, even when external cueing or redirection is provided.

• Short attention span, easy distractibility, inability to concentrate• Impulsive, impatient, low tolerance for pain or frustration• Uncooperative, resistant to care, demanding• Violent and or threatening violence toward people or property• Explosive and/or unpredictable anger• Rocking, rubbing, moaning or other self-stimulating behavior• Pulling at tubes, restraints, etc.• Wandering from treatment areas• Restlessness, pacing, excessive movement• Repetitive behaviors (motor and/or verbal)• Rapid, loud, or excessive talking• Sudden changes of mood• Easily initiated or excessive crying and/or laughter• Self-abusiveness (physical and/or verbal)

AGITATION SCORESevere = (≥36)

Moderate = (29-35)

Mild = (22–28)

None = (<22)

Critical Decisions in Emergency Medicine24

placed on a monitor, and an ECG should be obtained as soon as possible. Routine laboratory studies, including measurements of magnesium, CPK, venous blood gas, acetaminophen, and salicylate levels, should be considered.

Patients who present within 1 hour of ingestion and have a stable mental status or are intubated should receive activated charcoal. Whole bowel irrigation is a viable option for treating paliperidone ingestion, given the drug’s sustained-release preparation. If an anticholinergic toxidrome is present, physostigimine (1 to 2 mg IV every 60 to 90 minutes) can be administered safely to reduce delirium.62

Benzodiazepines can be used to treat both agitation and seizure prompted by AAP toxicity. As atypical antipsychotics are lipophilic, lipid emulsion therapy can be considered for severe overdose; it has proven effective in other lipid-soluble medication poisonings.63,64

Most AAPs are rapidly absorbed from the gastrointestinal tract, with the exception of paliperidone and aripiprazole; after a 6-hour observation period, patients generally can be cleared. (Note: see previous sections for the management of QTc interval prolongation, TdP, and NMS.)

CRITICAL DECISIONWhat specific characteristics and complications are associated with the most common second-generation antipsychotic agents?

A capsule summary of each drug follows. Olanzapine, the most common antipsychotic agent used in the emergency department, is discussed in more detail than the other medications.

Olanzapine (Zyprexa, Zydis)Dose: 5 to 10 mg PO (up to 20 mg/

day); or 10 mg IM every 4 hours for agitation (up to 30 mg/day)

Olanzapine is a serotonin type II (5-HT2) antagonist; the drug also antagonizes dopamine, muscarinic, alpha-1 and histamine H1 receptors. Although it is structurally similar to clozapine, it is less likely to cause seizures or agranulocytosis.

Its indications include schizophrenia and acute or mixed mania associated with bipolar I in adults. The medication is prescribed off-label for agitation, psychosis secondary to dementia, chronic pain, acute delirium, and pediatric schizophrenia; it also has been used to treat headaches.

When given intravenously as an adjunct to midazolam (2.5- to 5-mg boluses), olanzapine (5 mg) may provide more rapid sedation than midazolam alone.65 Of note, olanzapine carries the lowest risk of QT prolongation of all second-generation antipsychotics (thioridazine appears to pose the biggest danger).36,66

Other side effects include:• Extrapyramidal reactions• Elevated LFTs• Elevated CPK• Pancreatitis• Dizziness, drowsiness, headache,

weakness• Weight gain, metabolic

syndrome, diabetes mellitus• Orthostatic hypotension• Dry mouth, constipation• Hyperlipidemia• Sexual dysfunction• Chest and abdominal pain• Neuroleptic malignant

syndrome (the most serious reaction)

PrecautionsElderly patients taking olanzapine

with dementia-related psychosis may have an increased threat of death due to cardiovascular or infectious complications. Caution also must be used in patients with a high risk for suicide, and those with severe cardiac, renal or hepatic disease, and blood disorders. Sedation may be exacerbated in patients taking other sedatives. Olanzapine enters breastmilk and is not recommended for lactating women (pregnancy category C).

The following medications, which are summarized briefly, are more likely to be prescribed by psychiatrists.

Quetiapine (Seroquel)Dose: 50 mg (2x/day) up to 400 to

800 mg/dayQuetiapine is a dopamine, serotonin,

and adrenergic antagonist, and a potent antihistamine with clinically negligible anticholinergic properties. The drug has a high affinity for histamine and adrenergic α-1 receptors; tachycardia and hypotension are seen in cases of overdose.67

Other side effects include central nervous system (CNS) depression in overdose (eg, somnolence, dizziness), anticholinergic effects (eg, dry mouth), hyperglycemia, weight gain, priapism, QTc prolongation, and neuroleptic malignant syndrome.

n Short-acting intramuscular AAP formulations (eg, aripiprazole, olanzapine, and ziprasidone) are particularly suited and best studied for treating agitated patients.

n Agranulocytosis can occur up to 5 years after the initiation of clozapine; weekly CBC monitoring to detect early drops in WBCs is standard care for patients taking the drug.

n Central nervous system complications, including sedation, uniformly are seen in cases of AAP overdose.

n Schizophrenic patients taking antipsychotic medications are at an increased risk of sudden cardiac death.

n The classic presentation of drug-induced neuroleptic malignant syndrome is a triad of fever, muscle rigidity, and altered mental status.

April 2016 n Volume 30 Number 4 25

n Prescribing medications that can add to the effects of those the patient already may be taking.

n Failing to monitor the QRS and QT durations of a patient’s ECG in cases of suspected ingestion.

n Missing a diagnosis of agranulocytosis in any patient taking clozapine.

n Failing to monitor serum electrolytes and correct for hypomagnesemia and hypokalemia.

n Failing to monitor vital signs for evidence of neuroleptic malignant syndrome.

Ziprasidone (Geodon)Dose: 10 to 20 mg IM (20 to 80 mg

PO) 2x/dayPatients taking ziprasidone have a

lower incidence of EPS, likely due to the agent’s high serotonin-2A/dopamine-2 ratio. Marked drowsiness is seen with overdose; other side effects include nausea and weakness. QTc prolongation is more pronounced than with most other second-generation agents (mean prolongation of 15 to 20 ms over baseline QT).69 This generally is not an issue with low doses, unless there are underlying cardiac or metabolic disorders, or the drug is being used in combination with other medications known to prolong the QT interval68

Risperidone (Risperdal)Dose: 2 mg/day (maximum

16 mg/day) Side effects include sedation,

hypotension, weight gain, prolactin elevation, dystonic reactions, priapism, orthostasis, and NMS.

Aripiprazole (Abilify)Dose: 10 to 30 mg/daySedation can be significant in cases

of overdose. Aripiprazole does not have a clinically significant effect on cardiac conduction. Extrapyramidal symptoms are uncommon, and it poses a lower risk of hyperlipidemia and hyperglycemia than other antipsychotic agents. It is available for parenteral use. Adverse effects include anxiety, headache, insomnia, nausea/vomiting, and constipation.

Uncommon AgentsThe following agents will rarely be

prescribed by emergency physicians, and suggested doses have been omitted.

Clozapine (Clozaril, FazaClo)Developed in the 1970s, clozapine

was the first atypical antipsychotic on the market. The drug was reapproved by the FDA in 1989 because of its efficacy in treating refractory schizophrenia. Although it may be the most effective second-generation antipsychotic available, it carries a potential for serious toxicity.

Adverse effects include NMS, orthostatic hypotension, tachycardia,

seizures, myocarditis, diabetes, weight gain, and metabolic syndrome. It poses a low risk of extrapyramidal symptoms, but a greater risk of QTc prolongation than other agents. Leukopenia/granulocytopenia, and agranulocytosis may be seen (especially when used with carbamazepine or sulfonamide antibiotics); blood should be checked weekly after initiation. NMS, EPS, and tardive dyskinesia are uncommon.

Iloperidone (Fanapt)Adverse effects include orthostatic

hypotension, dizziness, dry mouth, sedation, priapism, and QTc prolongation (more than with other second-generation agents, except for ziprasidone). The drug carries relatively few extrapyramidal side effects.

Paliperadone (Invega)The major active metabolite of

risperidone, paliperadone is the only sustained-release atypical antipsychotic preparation available. Whole bowel irrigation may be considered in cases of overdose. Adverse effects include extrapyramidal symptoms (eg, Parkinsonism, dystonia, and akathisia), mild to moderate weight gain, and possible prolongation of the QT interval.

Lurasidone (Latuda)Adverse effects include akathisia,

nausea, extrapyramidal symptoms, and agitation; category B for use in pregnancy.

Asenapine (Saphris)Adverse effects include insomnia,

somnolence, dizziness, nausea/vomiting, weight gain, and EPS (akathisia).

Mirtazipine (Remeron)Adverse effects include postural

hypotension, insomnia, diabetes mellitus, weight gain, and priapism.

Brexpiprazole (Rexulti)The FDA recently approved

brexpiprazole for the treatment of schizophrenia, and as an adjunct to antidepressants for the treatment of major depression. It is an oral dopamine D2 and serotonin 5-HT1A antagonist, given once daily (typically, 2 to 4 mg).70 Adverse effects may include akathisia, weight gain, and somnolence. The drug appears to be well tolerated, and does not cause significant QTc prolongation.

SummarySecond-generation antipsychotics

have gained popularity in recent years due to their relative lack of extrapyramidal symptoms when compared to first-generation agents. Some emergency physicians may consider these drugs as first-line treatments in the management of agitated patients.

Although clozapine may be the most effective agent for treating psychosis, its risk for causing seizures and association with agranulocytosis has limited its use. The effect of these drugs on QTc interval prolongation should be considered, although the clinical significance of this complication

Critical Decisions in Emergency Medicine26

in patients without cardiac disease has not been established. Side effects are common, including weight gain, diabetes, hyperlipidemia, and hyperprolactinemia.

REFERENCES1. Lai MW, Klein-Schwartz W, Rodgers GC, et al.

2005 Annual Report of the American Association of Poison Control Centers’ national poisoning and exposure database. Clin Toxicol (Phila). 2006;44(6-7):803-932.

2. Buckley NA, Whyte IM, Dawson AH. Cardiotoxicity more common in thioridazine overdose than with other neuroleptics. J Toxicol Clin Toxicol. 1995;33(3):199-204.

3. Denvir MA, Sood A, Dow R, et al. Thioridazine, diarrhoea and torsades de pointe. J R Soc Med. 1998;91(3):145-147.

4. Berling I, Isbister GK. Prolonged QT risk assessment in antipsychotic overdose using the QT nomogram. Ann Emerg Med. 2015;66(2):154-164.

5. Sharma ND, Rosman HS, Padhi ID, Tisdale JE. Torsades de pointes associated with intravenous haloperidol in critically ill patients. Am J Cardiol. 1998;81(2):238-240.

6. Thase ME, Montgomery S, Papakostas GI, et al. Quetiapine XR monotherapy in major depressive disorder: a pooled analysis to assess the influence of baseline severity on efficacy. Int Clin Psychopharmacol. 2013;28(3):113-120.

7. Gerhard T, Akincigil A, Correll CU, et al. National trends in second-generation antipsychotic augmentation for nonpsychotic depression. J Clin Psychiatry. 2014;75(5):490-497.

8. Yatham LN, Kennedy SH, Parikh SV, et al. Canadian Network for Mood and Anxiety Treatments (CANMAT) and International Society for Bipolar Disorders (ISBD) collaborative update of CANMAT guidelines for the management of patients with bipolar disorder: update 2013. Bipolar Disord. 2013;15(1):1-44.

9. Keck PE Jr, Calabrese JR, McIntyre RS, et al. Aripiprazole monotherapy for maintenance therapy in bipolar I disorder: a 100-week, double-blind study versus placebo. J Clin Psychiatry. 2007;68(10):1480-1491.

10. Cipriani A, Rendell J, Geddes JR. Olanzapine in the long-term treatment of bipolar disorder: a systematic review and meta-analysis. J Psychopharmacol. 2010;24(12):1729-1738.

11. McDougle CJ, Epperson CN, Pelton GH, et al. A double-blind, placebo-controlled study of risperidone addition in serotonin reuptake inhibitor-refractory obsessive-compulsive disorder. Arch Gen Psychiatry. 2000;57(8):794-801.

12. Sayyah M, Sayyah M, Boostani H, et al. Effects of aripiprazole augmentation in treatment-resistant obsessive-compulsive disorder (a double blind clinical trial). Depress Anxiety. 2012;29(10):850-854.

13. Nagaraj R, Singhi P, Malhi P. Risperidone in children with autism: randomized, placebo-controlled, double-blind study. J Child Neurol. 2006;21(6):450-455.

14. Findling RL, Mankoski R, Timko K, et al. A randomized controlled trial investigating the

safety and efficacy of aripiprazole in the long-term maintenance treatment of pediatric patients with irritability associated with autistic disorder. J Clin Psychiatry. 2014;75(1):22-30.

15. McDougle CJ, Scahill L, Aman MG, et al. Risperidone for the core symptom domains of autism: results from the study by the autism network of the research units on pediatric psychopharmacology. Am J Psychiatry. 2005;162(6):1142-1148.

16. Larkin GL, Claassen CA, Emond JA, et al. Trends in U.S. emergency department visits for mental health conditions, 1992-2001. Psychiatric Serv. 2005;56(6):671-677.

17. Allen MH. Managing the agitated psychotic patient: a reappraisal of the evidence. J Clin Psychiatry. 2000;61 Suppl 14:11-20.

18. Buckley P, Citrome L, Nichita C, Vitacco M. Psychopharmacology of aggression in schizophrenia. Schizophr Bull. 2011;37(5):930-936.

19. Daniel DG, Potkin SG, Reeves KR, et al. Intramuscular (IM) ziprasidone 20 mg is effective in reducing acute agitation associated with psychosis: a double-blind, randomized trial. Psychopharmacology (Berl). 2001;155(2):128-134.

20. Lesem MD, Zajecka JM, Swift RH, et al. Intramuscular ziprasidone, 2 mg versus 10 mg, in the short-term management of agitated psychotic patients. J Clin Psychiatry. 2001;62(1):12-18.

21. Preval H, Klotz SG, Southard R, Francis A. Rapid-acting IM ziprasidone in a psychiatric emergency service: a naturalistic study. Gen Hosp Psychiatry. 2005;27(2):140-144.

CASE RESOLUTIONS

■ CASE ONEThe fatigued middle-aged man’s

work of breathing continued to worsen as his heart rate increased to 125 beats per minute, and blood pressure dropped to 95/45. His oxygen saturation fell to 85%, but improved to 92% on 4 liters nasal cannula. A review of the patient’s chart revealed he recently had been down-titrated off of risperidone, and another antipsychotic had been prescribed 1 week prior to presentation. Laboratory tests revealed a troponin level of 13.37 with an N-terminal pro b-type natriuretic peptide of 7887; a chest x-ray showed bilateral pulmonary edema.

The patient went into cardiac arrest, and became unresponsive and pulseless. He was successfully defibrillated with one cycle of CPR, but remained unresponsive and was intubated for airway protection without complications. A repeat ECG showed T-wave inversions in the lateral leads, and new ST-segment depressions. He was admitted to the catheterization lab, where he was found to have an ejection fraction of 28%, but no coronary artery

disease. He received an automatic implantable cardioverter device, and was discharged neurologically intact with a diagnosis of clozapine myocarditis.

■ CASE TWOThe suicidal woman’s laboratory

tests revealed a slightly low potassium level (3.2 mEq/L), and a serum magnesium of 1.3; liver function tests were normal, and acetaminophen and acetylsalicylic acid levels were undetectable. An initial ECG showed a QTc interval sinus rhythm of 515 msec. This narrowed, following further magnesium replacement and the administration of potassium; and she sustained no further dysrhythmias during her in-patient stay. Additional questioning revealed that the patient had been taking erythromycin for bronchitis, and she admitted to participation in a methadone program following 6 years of heroin abuse.

The clinician ascertained that the likely culprit behind the episode of torsades de pointes was QTc prolongation spurred by the patient’s erythromycin and methadone use —

and compounded by the administration of ondansetron (rather than from risperidol). These agents were discontinued; however, she remained on medication to treat her psychosis.

■ CASE THREEDuring the consultation, the

combative, paranoid patient became diaphoretic and febrile (39.7◦C [103.6◦F]), and lorazepam was administered to allow further assessment. Additional laboratory testing revealed normal electrolytes and a total white blood count of 1,200 with no detectable neutrophils. A search of the patient’s home by family members revealed a prescription for clozapine. Blood cultures were drawn, a complete sepsis workup was ordered, and treatment was initiated.

After a stormy ICU course, during which the patient became severely hypotensive, he was transferred to an extended care facility. His mentation was clear on discharge, and his altered behavior on admission was attributed to sepsis compounded by underlying psychosis.

April 2016 n Volume 30 Number 4 27

22. Wright P, Birkett M, David SR, et al. Double-blind, placebo-controlled comparison of intramuscular olanzapine and intramuscular haloperidol in the treatment of acute agitation in schizophrenia. Am J Psychiatry. 2001;158(7):1149-1151.

23. Meehan KM, Wang H, David SR, et al. Comparison of rapidly acting intramuscular olanzapine, lorazepam, and placebo: a double-blind, randomized study in acutely agitated patients with dementia. Neuropsychopharmacology. 2002;26(4):494-504.

24. Lukens TW, Wolf SJ, Edlow JA, et al. Clinical policy: critical issues in the diagnosis and management of the adult psychiatric patient in the emergency department. Ann Emerg Med. 2006;47(1):79-99.

25. Meltzer HY, Alphs L, Green AI, et al. Clozapine treatment for suicidality in schizophrenia: International Suicide Prevention Trial (InterSePT). Arch Gen Psychiatry. 2003;60(1):82-91.

26. Tiihonen J, Lönngvist J, Wahlbeck K, et al. 11-year follow-up of mortality in patients with schizophrenia: a population-based cohort study (FIN11 study). Lancet. 2009;374(9690):620-627.

27. Minns AB, Clark RF. Toxicology and overdose of atypical antipsychotics. J Emerg Med. 2012;43(5):906-913.

28. Velayudhan R, Kakkan S. Late onset clozapine induced agranulocytosis. Indian J Psychol Med. 2014;36(4):425-427.

29. Merrill DB, Ahmari SE, Bradford JM, Lieberman JA. Myocarditis during clozapine treatment. Am J Psychiatry. 2006;163(2):204-208.

30. Killian JG, Kerr K, Lawrence C, Celermajer DS. Myocarditis and cardiomyopathy associated with clozapine. Lancet. 1999;354(9193):1841-1845.

31. La Grenade L, Graham D, Trontell A. Myocarditis and cardiomyopathy associated with clozapine use in the United States. N Engl J Med. 2001;345(3):224-225.

32. Burian J, Buser P, Eriksson U. Myocarditis: the immunologist’s view on pathogenesis and treatment. Swiss Med Wkly. 2005;135(25-26):359-364.

33. Berman BD. Neuroleptic malignant syndrome: a review for neurohospitalists. Neurohospitalist. 2011;1(1):41-47. doi:10.1177/1941875210386491.

34. Williams AM, Park SH. Seizure associated with clozapine: incidence, etiology, and management. CNS Drugs. 2015;29(2):101-111.

35. Nielsen J, Graff C, Kanters JK, et al. Assessing QT interval prolongation and its associated risks with antipsychotics. CNS Drugs. 2011;25(6):473-490.

36. Nachimuthu S, Assar MD, Schussler JM. Drug-induced QT interval prolongation: mechanisms and clinical management. Ther Adv Drug Saf. 2012;3(5):241-253. doi:10.1177/2042098612454283.

37. Wenzel-Seifert K, Wittmann M, Haen E. QTc prolongation by psychotropic drugs and the risk of torsade de pointes. Dtsch Arztebl Int. 2011;108(41):687-693.

38. Fazio G, Vernuccio F, Grutta G, Re GL. Drugs to be avoided in patients with long QT syndrome: Focus on the anaesthesiological management. World J Cardiol. 2013;5(4):87-93. doi:10.4330/wjc.v5.i4

39. Chan A, Isbister GK, Kirkpatrick CM, Dufful SB. Drug-induced QT prolongation and torsades de pointes: evaluation of a QT nomogram. QJM. 2007;100(10):609-615.

40. Titier K, Canal M, Déridet E, et al. Determination of myocardium to plasma concentration rations of five antipsychotic drugs: comparison with their ability to induce arrhythmia and sudden death in clinical practice. Toxicol Appl Pharmacol. 2004;199(1):52-60.

41. Ngo A, Ciranni M, Olsen KR. Acute quetiapine overdose in adults: a 5-year retrospective case series. 2008;52(5):541-547.

42. Sicouri S, Antzelevitch C. Sudden cardiac death secondary to antidepressant and antipsychotic drugs. Expert Opin Drug Saf. 2008;7(2):181-194.

43. Liperoti R, Gambassi G, Lapane KL, et al. Conventional and atypical antipsychotics and the risk of hospitalization for ventricular arrhythmias or cardiac arrest. Arch Intern Med. 2005;165(6):696-701

44. Glassman AH, Bigger JT Jr. Antipsychotic drugs: prolonged QTc interval, torsade de pointes, and sudden death. Am J Psychiatry. 2001;158(11):1774-1782.

45. Stroup TS, Lieberman JA, McEvoy JP, et al. Effectiveness of olanzapine, quetiapine, and risperidone in patients with chronic schizophrenia after discontinuing perphenazine: a CATIE study. Am J Psychiatry. 2007;164(3):415-427.

46. Tzivoni D, Banai S, Schuger C, et al. Treatment of torsade de pointes with magnesium sulfate. Circulation. 1988;77(2):392-397.

47. Khan IA. Long QT syndrome: diagnosis and management. Am Heart J. 2002;143(1):7-14.

48. DiSegni E, Klein HO, David D, et al. Overdrive pacing in quinidine syncope and other long QT-interval syndromes. Arch Intern Med. 1980;140(8):1036-1040.

49. Keren A, Tzivoni D, Gavish D, et al. Etiology, warning signs and therapy of torsade de pointes. A study of 10 patients. Circulation. 1981;64(6): 1167-74.

50. Takahashi N, Ito M, Inoue T, et al. Torsades de pointes associated with acquired long QT syndrome: observation of 7 cases. J Cardiol. 1993; 23(1):99-106.

51. Choy AM, Lang CC, Chomsky DM, et al. Normalization of acquired QT prolongation in humans by intravenous potassium. Circulation. 1997;96(7):2149-2154.

52. Tan HH, Hoppe J, Heard K. A systematic review of cardiovascular effects after atypical antipsychotic medication overdose. Am J Emerg Med. 2009;27(5):607-616.

53. Bak M, Fransen A, Janssen J, et al. Almost all antipsychotics result in weight gain: a meta-analysis. PLoS One. 2014;9(4):e94112. doi:10.1371/journal.pone.0094112.

54. Bobes J, Rejas J, Garcia-Garcia M, et al. Weight gain in patients with schizophrenia treated with risperidone, olanzapine, quetiapine or haloperidol: results of the EIRE study. Schizophr Res. 2003;62(1-2):77-88.

55. Leslie DL, Rosenheck RA. Incidence of newly diagnosed diabetes attributable to atypical antipsychotic medications. Am J Psychiatry. 2004;161(9):1709-1711.

56. Roerig JL, Steffen KJ, Mitchell JE. Atypical antipsychotic-induced weight gain: insights into mechanisms of action. CNS Drugs. 2011;25(12):1035-1059.

57. Bronstein AC, Spyker DA, Cantilena LR Jr, et al. 2009 Annual report of the American Association of Poison Control Centers’ National Poison Data System (NPDS): 27th annual report. Clin Toxicol (Phila). 2010;48(10):979-1178.

58. Reilly JG, Ayis SA, Ferrier IN, et al. QTc-interval abnormalities and psychotropic drug therapy in psychiatric patients. Lancet. 2000;355(9209):1048-1052.

59. Page CB, Calver LA, Isbister GK. Risperidone overdose causes extrapyramidal effects but not cardiac toxicity. J Clin Psychopharmacol. 2010;30(4):387-390.

60. Akyol A, Senel AC, Ulusoy H, et al. Delayed respiratory depression after risperidone overdose. Anesth Analg. 2005;101(5):1490-1491.

61. Waring WS, Wrate j, Bateman DN. Olanzapine overdose is associated with acute muscle toxicity. Hum Exp Toxicol. 2006;25(12):735-740.

62. Schneir AB, Offerman SR, Ly BT, et al. Complications of diagnostic physostigmine administration to emergency department patients. Ann Emerg Med. 2003;42(1):14-19.

63. Grace RF, Newell SD. Paradoxical and severe hypotension in response to adrenaline infusions in massive quetiapine overdose: the case for lipid rescue. Crit Care Resusc. 2009;11(2):162.

64. Brull SJ. Lipid emulsion for the treatment of local anesthetic toxicity: patient safety implications. Anesth Analg. 2008;106(5):1337-1339.

65. Chan EW, Taylor DM, Knott JC, et al. Intravenous droperidol or olanzapine as an adjunct to midazolam for the acutely agitated patient: a multicenter, randomized, double-blind, placebo-controlled trial. Ann Emerg Med. 2013;61(1):72-81.

66. Harrigan EP, Miceli JJ, Anziano R, et al. A randomized evaluation of the effects of six antipsychotic agents on QTc, in the absence and presence of metabolic inhibition. J Clin Psychopharmacol. 2004;24(1):62-69.

67. Hustey FM. Acute quetiapine poisoning. J Emerg Med. 1999;17(6):995-997.

68. Witsil JC, Zell-Kanter M, Mycyk MB. Single-dose ziprasidone associated with QT interval prolongation. Am J Emerg Med. 2012;30(5):837.

69. Carnahan RM, Lund BC, Perry PJ. Ziprasidone, a new atypical antipsychotic drug. Pharmacotherapy. 2001;21(6):717-730.

70. Brexpiprazole (Rexulti) for schizophrenia and depression. Med Lett Drugs Ther. 2015;57(1475):116-118.

ADDITIONAL READINGNachimuthu S, Assar MD, Schussler JM. Drug-induced

QT interval prolongation: mechanisms and clinical management. Ther Adv Drug Saf. 2012;3(5):241-253.

Fazio G, Vernuccio F, Grutta G, Re GL. Drugs to be avoided in patients with long QT syndrome: Focus on the anaesthesiological management. World J Cardiol. 2013;5(4):87-93.

Bogner JA, Corrigan JD, Bode RK, Heinemann AW. Rating scale analysis of the Agitated Behavior Scale. J Head Trauma Rehabil. 2000;15(1):656-669.

Bogner JA, Corrigan JD, Stange M, Rabold D. Reliability of the Agitated Behavior Scale. J Head Trauma Rehabil. 1999;14(1):91-96.

Critical Decisions in Emergency Medicine28

The Critical Image

CASE A 53-year-old man with no past medical history awakens from a 2-hour nap with aphasia and right hemiparesis. He presents to a community emergency department within 3 hours of his last-known normal neurologic status, and is given IV tPA for ischemic stroke following a noncontrast head CT, which demonstrates no evidence of hemorrhage. He then is transferred to the emergency department of a regional stroke center. Vital signs are blood

By Joshua S. Broder, MD, FACEPDr. Broder is an associate professor and the residency program director in the Division of Emergency Medicine at Duke University Medical Center in Durham, North Carolina.

REFERENCES1. Wardlaw JM, Mielke O. Early signs of brain infarction at CT: observer reliability and outcome after thrombolytic treatment—systematic review. Radiology 2005;235:444-453.

2. Lev MH, Farkas J, Gemmete JJ, et al. Acute stroke: improved nonenhanced CT detection—benefits of soft-copy interpretation by using variable window width and center level settings. Radiology 1999;213:150-155.

3. Turner PJ, Holdsworth G. Commentary. CT stroke window settings: an unfortunate misleading misnomer? Br J Radiol 2011;84:1061-1066.

4. Berkhemer OA, Fransen PS, Beumer D, et al. A randomized trial of intraarterial treatment for acute ischemic stroke. N Engl J Med 2015;372:11-20.

5. Berkhemer OA, Majoie CB, Dippel DW. Intraarterial treatment for acute ischemic stroke. N Engl J Med 2015;372:1178-1179.

6. Broderick JP, Berkhemer OA, Palesch YY, et al. Endovascular Therapy Is Effective and Safe for Patients With Severe Ischemic Stroke: Pooled Analysis of Interventional Management of Stroke III and Multicenter Randomized Clinical Trial of Endovascular Therapy for Acute Ischemic Stroke in the Netherlands Data. Stroke 2015;46:3416-3422.

7. Rozeman AD, Wermer MJ, Vos JA, et al. Evolution of intra-arterial therapy for acute ischemic stroke in the Netherlands: MR CLEAN pretrial experience. J Stroke Cerebrovasc Dis 2016;25:115-121.

8. Kappelhof M, Marquering HA, Berkhemer OA, et al. Intra-arterial treatment of patients with acute ischemic stroke and internal carotid artery occlusion: a literature review. J Neurointerv Surg 2015;7:8-15.

pressure 112/73 mmHg, pulse 64, respiratory rate 18, temperature 37.3°C (99.1°F), and oxygen saturation 100% on room air.The patient is awake, but has both expressive and receptive aphasia. His right upper and lower extremities show 3/5 strength,

and his right face appears weak. Based on these deficits, his NIH Stroke Scale is calculated at 12. The remainder of his general physical examination is normal, and an ECG demonstrates a normal sinus rhythm. A noncontrast head CT is performed, followed by CT angiography of the brain and neck.

A. Noncontrast CT viewed on a common “brain window” setting, with a center value of 40 HU and a width of 80 HU. An area of hypodensity representing ischemic stroke is present, but could easily be overlooked.

B. The same noncontrast CT slice as in A, now shown with a “stroke window” setting (center value 40 HU, and width 40 HU). The area of hypodensity is accentuated.

C and D. Axial neck CT angiography images, proximal and distal to the point of internal carotid artery (ICA) occlusion, respectively. Proximal to the occlusion, the ICA fills with contrast, whereas distally it does not.

E. 3D CT reconstruction reveals the abrupt cutoff of the ICA, indicating occlusion

BA Area of infarction is accentuated by narrow “stroke window” setting

April 2016 n Volume 30 Number 4 29

CASE RESOLUTIONThe patient underwent endovascular therapy with stent placement in the left proximal internal carotid artery. At discharge, his motor examination had normalized, and he displayed some mild residual expressive and receptive aphasia.

D

C

KEY POINTSn The primary purpose of noncontrast

head CT in the initial evaluation of potential stroke is to identify intracranial hemorrhage contraindicating adminis-tration of tPA, or pinpoint alternative diagnoses such as intracranial mass lesions. Confirmation of ischemic stroke is less important, as the initial head CT may be normal; however, early findings of this pathology on noncontrast head CT can include loss of gray-white matter differentiation and localized edema.1

n Identification of ischemic stroke can be facilitated by appropriate use of CT windows, which are defined by a center value and a window width, both measured in Hounsfield units (HU). These values de-scribe the distribution of the gray shades assigned to tissue densities. Tissues with densities below the lower limit of the window are assigned black, while those above the upper limit of the window are

assigned white; the intermediate densities within the window appear varying shades of gray. A common brain window setting has a center of 40 HU and a width of 80 HU, al-though no standard “brain window” is defined. With these settings, tissue with a density of 40 HU (the center value) would appear gray, while the window width of 80 indicates that black and white shades would occur at values of 0 HU (center minus ½ window width) and 80 HU (center plus ½ window width), respectively.

n Narrowing the window width improves the ability to distinguish subtle changes in tissue density within the window range, enhanc-ing brain soft-tissue contrast at the expense of losing all discriminatory capability outside of that window. Narrow user-determined “stroke windows” have been shown to im-prove detection of ischemic stroke, compared with a window setting of center 20 HU and width 80 HU.2 A common stroke window setting has a center value of 40 HU and a width of 40 HU, although individual users may prefer different settings. This setting can be helpful in detecting subtle brain parenchyma lesions other than stroke.3 PACS (Picture Archiving and Communication System, the digital image viewing system) users can alter the center and width values manually with menu options or by

manipulating the CT images with a mouse, although individual software features vary.

n Recent studies, including the Multicenter Randomized Clinical Trial of Endovascular Treatment for Acute Ischemic Stroke in the Netherlands (MR CLEAN), have demonstrated improved stroke functional outcomes when IV tPA is followed by intra-arterial therapy (ie, intra-arterial thrombolysis, mechanical treatment, or both) within 6 hours of stroke onset for patients with proximal arterial occlusion of the anterior cerebral circulation, based on neuroimaging such as CT angiography.4-7 In most cases, noncontrast head CT remains the initial study of choice to rule out hemorrhage.

n For patients with extracranial internal carotid artery occlusion, stenting and mechanical thrombectomy are associated with better recanalization and improved clinical outcomes compared with intra-arterial thrombolysis.8

E

Normal right internal carotid artery fills with contrast

Proximal to the level of occlusion, the left internal carotid artery fills with contrast

Distal to the level of occlusion, the left internal carotid artery does not fill with contrast

Normal right internal carotid artery fills with contrast

Distal to the level of occlusion, the left

internal carotid artery does

not fill with contrast

Critical Decisions in Emergency Medicine30

CME QUESTIONS

1 Which of the following local anesthetic agents has the shortest duration of action?A. 2-chloroprocaineB. BupivacaineC. LidocaineD. Mepivacaine

2 What is the advantage of using an in-plane approach as opposed to an out-of-plane approach for placement of an ultrasound-guided nerve block?A. Ability to visualize all surrounding structures B. Ability to visualize entire course of the needleC. Easier to place on most patientsD. Less risk of puncturing surrounding structures

3 What is the biggest challenge when using an out-of-plane approach?A. Greater incidence of anisotropy B. Placement of probe on patientC. Visualizing adjacent structuresD. Visualizing the needle tip

4 Which of the following is a contraindication to placement of a peripheral nerve block?A. Altered mental statusB. Anticoagulation useC. Distracting injuryD. Patient age >65 years old

5 A 95-year-old man with a history of atrial fibrillation and diabetes presents with a right hip fracture following a mechanical fall. His evaluation reveals no other injuries. The orthopedic team reports a plan for operative repair in approximately 15 hours. The patient has no medication allergies. Which of the following local anesthetic agents is most appropriate in this case?A. 2-chloroprocaine B. BupivacaineC. LidocaineD. Mepivacaine

6 Which term describes the property of peripheral nerves changing their “honeycomb” appearance with subtle movements of the ultrasound transducer?A. AnisotropyB. Edge artifact C. Mirror-artifactD. Reverberation

7 Which of the following is a potential complication of an intravascular injection of local anesthetic?A. Chronic paresthesiasB. Infection C. Mechanical nerve damageD. Seizure

8 A 55-year-old woman begins to have a seizure during a peripheral nerve block. The provider failed to aspirate the syringe prior to injecting 30 mg of 0.5% bupivacaine. Which of the following medications is the next most appropriate therapy?A. 20% IV fat emulsionB. Hypertonic saline C. LidocaineD. Sodium bicarbonate

9 A 35-year-old man with a history of intravenous drug abuse presents with a painful abscess on the sole of his foot. Which of the following peripheral blocks is most appropriate for incision and drainage?A. Fascia iliaca blockB. Median nerve blockC. Posterior tibial nerve blockD. Ulnar nerve block

Qualified, paid subscribers to Critical Decisions in Emergency Medicine may receive CME certificates for up to 5 ACEP Category I credits, 5 AMA PRA Category 1 Credits™, and 5 AOA Category 2-B credits for completing this activity in its entirety. Submit your answers online at acep.org/newcriticaldecisionstesting; a score of 75% or better is required. You may receive credit for completing the CME activity any time within three years of its publication date. Answers to this month’s questions will be published in next month’s issue.

Reviewed by Lynn Roppolo, MD, FACEP

April 2016 n Volume 30 Number 4 31

ANSWER KEY FOR MARCH 2016, VOLUME 30, NUMBER 3

1 2 3 4 5 6 7 8 9 10 11 12 13 14 15 16 17 18 19 20B C B B A D B B B D D B D D A D B D C D

When performing a fascia iliaca block, where should the needle tip be located when injecting local anesthetic? A. Deep to the fascia iliacaB. Deep to the iliacus muscle C. Medial to the femoral veinD. Superficial to the fascia lata

Which of the following side effects constitute extrapyramidal symptoms?A. Akathisia, Parkinsonism, dystoniaB. Akathisia, Parkinsonism, hyporeflexiaC. Hyperreflexia, Parkinsonism, akathisiaD. Myoclonus, hyperreflexia, akathisia

Which complications uniformly are seen in cases of atypical antipsychotic agent overdose?A. Central nervous system effects, including sedationB. Elevated creatine phosphokinaseC. Leukopenia/granulocytopeniaD. QT prolongation

Which agents are used to treat acute agitation?A. Clozapine, olanzapine B. Olanzapine, ziprasidoneC. Risperidone, iloperidoneD. Risperidone, paliperidone

What complication of clozapine requires weekly monitoring?A. AgranulocytosisB. MyocarditisC. QTc prolongationD. Weight gain

What is the incidence of clozapine-induced agran-ulocytosis, and when do 75% of these cases occur?A. 1%, within 6 monthsB. 1%, within 1 monthC. 5%, within 1 weekD. 6%, within 24 hours

The classic triad of neuroleptic malignant syndrome includes fever, altered mental status, and which of the following symptoms?A. Muscle relaxationB. Muscle rigidityC. SeizureD. Vomiting

What effect does dantrolene have when used for the treatment of neuroleptic malignant syndrome?A. It can improve overall morbidityB. It can improve overall mortalityC. It has no effect on symptom durationD. It may decrease symptom duration

Which of the following risk factors are associated with prolonged QTc intervals?A. Elderly, female, hypokalemiaB. Elderly, male, hyponatremia C. Young, female, hyponatremiaD. Young, male, hypokalemia

Which of the following protocols can be used for the management of prolonged QTc intervals?A. Cardiac pacing, isoproterenolB. Carotid massage, labetalolC. Glucagon, leg liftD. Synchronized cardioversion, clonidine

What atypical antipsychotic agents carry the greatest risk for metabolic syndrome?A. Olanzapine, clozapineB. Quetiapine, lurasidoneC. Ziprasidone, aripiprazoleD. Ziprasidone, olanzapine

10

11

12

13

14

15

16

17

19

18

20

Drug Box Tox BoxDIPHENHYDRAMINE POISONINGBy Bryan Corbett, MD, UC San Diego Medical Center, San Diego, CA

COLCHICINEBy Hannah Malashock, MD, and Frank Lovecchio, DO, MPH, Maricopa and Banner University Medical Centers, Phoenix, AZ

Colchicine is an anti-inflammatory agent indicated for the treatment of gout and familial Mediterranean fever, and has gained popularity for off-label use in patients with pericarditis.

Mechanism of Action

Prevents inflammation by inhibiting microtubule polymerization (preventing leukocyte activation, degranulation, and migration). It also may interfere with the activation of inflammatory mediators.

Indications FDA-approved to treat acute gout; can be used as prophylaxis in between. Also indicated for familial Mediterranean fever in patients ≥4 yo. Relevant off-label uses include the treatment of both acute and chronic pericarditis in adults. As an adjunct to NSAID therapy, it may reduce symptoms and decrease the rate of recurrence (when compared to NSAID therapy alone).

Dosing

Off-Label Dosing

Acute gout flare: Initial dose of 1.2 mg PO, followed by a single dose (0.6 mg) 1 hour later Gout prophylaxis: 0.6 mg PO/1-2x daily for 3-6 months Familial Mediterranean fever: 1.2-2.4 mg PO daily, divided into 1-2 dosesPericarditis (acute): 0.5 mg PO/2x day for 3 months (0.5 mg daily if <70 kg)Pericarditis (recurrent): 0.5 mg PO/2x day for 6 months (0.5 mg daily if <70 kg); may consider loading dose (0.5-1.2 mg every 12 hours) on day 1, followed by above dosing

Side Effects

Most common: Diarrhea, abdominal pain, nausea, emesis, reversible peripheral neuropathy Rare (<1%): Bone marrow suppression, hepatotoxicity, myotoxicity, rhabdomyolysis, severe cutaneous eruptions

Consider - ations

Contraindications: Impaired hepatic or renal function Precautions: Multiple interactions with P-glycoprotein or CYP3A4 inhibitors. Check for specific drug-drug interactions prior to use. Pregnancy and lactation: Class C, crosses human placenta and breastmilk; use with caution in women who are breastfeeding.

PresentationDiphenhydramine is a ubiquitous medication in over-the-counter cough and cold preparations. An antihistamine (H1-blocker), it frequently is used to treat allergies and induce sleep. It is sometimes abused recreationally owing to its anti-muscarinic effects and easy accessibility; poisonings are common. Toxicity presents as an anti-muscarinic toxidrome of mydriasis, tachycardia, dry mucous membranes, decreased or absent sweating, urinary retention, decreased gastrointestinal motility, and altered senso-rium. The most concerning complication is QRS widening with subsequent ventricular dysrhythmias and seizures; this troubling effect is caused by the drug’s propensity to block fast voltage-gated sodium (Na) channels.ToxicitySeen with ingestions of 8-10 mg/kg. Doses of 20-40 mg/kg can be fatal.Laboratory Evaluation• Diagnosis is based on the history of ingestion, and anti-muscarinic signs. • Diphenhydramine levels are not readily available.• An ECG is essential to assess QRS duration. Values >100 msec are

abnormal; some clinicians do not intervene until values reach 120 msec.• A prominent terminal R wave in lead aVR can be seen due to Na-

channel blockade.Treatment• IV fluids are the first line for hypotension.• Na bicarbonate should be administered for QRS prolongation

(1-2 ampule IV boluses; repeat as needed, taking care to monitor pH).• Physostigmine IV (1 mg for adults; 0.02 mg/kg for peds) over 1 minute

can reverse delirium; repeat dose if no response. Avoid in cases of QRS widening.

• Lidocaine (1 mg/kg) is recommended for ventricular dysrhythmias recalcitrant to defibrillation or cardioversion.

• Intralipid therapy can be given for persistent hypotension and cardio-vascular collapse (1.5 mL/kg of 20% lipid emulsion bolus over 1 min; infuse at 0.25 mL/kg/hr). Repeat bolus for persistent hypotension and double infusion rate.

Decontamination/EliminationActivated charcoal is useful if given within 1 hour of large ingestions. Be wary of altered mental status and seizure. May consider whole bowel irrigation in cases of ileus. Hemodialysis and other methods of extracorporeal drug removal are ineffective.DispositionPatients with a QRS duration <100 msec and no delirium may be discharged after adequate observation (usually 4-6 hours).

Reviewed by Christian Tomaszewski, MD, MS MBA, FACEP


Recommended